1532

Since I'm not that good at (as I like to call it) 'die-hard-mathematics', I've always liked concepts like the golden ratio or the dragon curve, which are easy to understand and explain but are mathematically beautiful at the same time.

Do you know of any other concepts like these?

blademan9999
  • 603
  • 5
  • 21
RBS
  • 811
  • 3
  • 11
  • 19
  • 41
    It looks like mathpop or demand for math entertainment) – rook Apr 02 '14 at 12:59
  • 13
    Something that has always annoyed me is irrational numbers. They are easy to understand (like the proof for $\sqrt{2}$), but not being able to have a rational representation just irks me. And don't even get me started on transcendentals like $\pi$ and $e$. – Cole Tobin Apr 04 '14 at 16:12
  • 18
    @ColeJohnson the 'transcendentality" is the beauty of it! – Guy Apr 04 '14 at 17:07
  • 2
    @ColeJohnson Have you seen [Cantor's diagonal argument](http://en.wikipedia.org/wiki/Cantor%27s_diagonal_argument) which, given any enumerable set (such as the rational numbers), shows how to construct an element (an irrational number) that does not belong to that set. It can be fairly easily adapted to construct an irrational number between any two rational numbers. – chepner Apr 06 '14 at 15:22
  • 2
    Obligatory mention: _Donald Duck in Mathemagic Land_. (I believe there's a complete copy on YouTube.) – keshlam Apr 07 '14 at 02:08
  • 2
    One of the proofs of the geometric series summ at: http://www41.homepage.villanova.edu/robert.styer/Bouncingball/geometric_series_3.htm (see Fig. 4), it may interest you. – NoChance May 09 '14 at 20:41
  • 11
    There is a considerable overlap with http://mathoverflow.net/questions/8846/proofs-without-words – Martin Brandenburg Jun 20 '14 at 22:13
  • 2
    @LTS https://www.facebook.com/michiokaku/posts/10152632645341203 – The Guy with The Hat Aug 29 '14 at 17:08
  • 13
    @TheGuywithTheHat That's the reason for the second spike of visits, on August 27. The comment by LTS is from April; back then the traffic was driven by [Ycombinator](https://news.ycombinator.com/item?id=7543691). As a result, this same question made both [April 7 and August 27](http://chat.stackexchange.com/transcript/36?m=17402105#17402105) the two days with most visits to the site. –  Aug 29 '14 at 18:37
  • 2
    May want to look at vihart's stuff. – Please stop being evil Apr 08 '15 at 02:29
  • 2
    Fibonacci's spiral? – DVD Nov 10 '16 at 22:33

86 Answers86

1085

circle trig animation

I think if you look at this animation and think about it long enough, you'll understand:

  • Why circles and right-angle triangles and angles are all related.
  • Why sine is "opposite over hypotenuse" and so on.
  • Why cosine is simply sine but offset by $\frac{\pi}{2}$ radians.
AryanSonwatikar
  • 2,690
  • 1
  • 12
  • 36
congusbongus
  • 581
  • 1
  • 8
  • 12
  • 7
    @congusbongus I'm not sure how this was generated, but I wonder is there is a way to slow down and or stop the animation? +1 Fabulous. – joeA Apr 01 '14 at 21:47
  • 22
    @joeA: it's not as smooth as regenerating at a higher framerate, but gfycat.com allows one to view gifs at different speeds: http://gfycat.com/TintedWatchfulAxisdeer#?speed=0.25 – Max Apr 02 '14 at 20:51
  • 4
    The trigonometric perpetual machine... – Matemáticos Chibchas Apr 03 '14 at 16:28
  • 265
    If high school math just used a fraction of the resources here, we'd have way more mathematicians. – user148298 Apr 03 '14 at 17:00
  • 1
    I want to see a version that has the *{0,0}* point for all the charts aligned on the center of the circle... would they still line up? – Patrick M Apr 03 '14 at 20:44
  • 32
    Is the source of this animation available? (It looks like it's in $\mathrm\TeX$.) – bb010g Apr 03 '14 at 23:31
  • I never understood sin and cos until I learned their relations to circles. Then, this was exactly what I pictured. +1 – Justin Apr 05 '14 at 17:42
  • 8
    To me, it is quite confusing. I doubt I would learn much from this. Beautiful, yes, but the best in here?! – Rolazaro Azeveires Apr 05 '14 at 20:54
  • 3
    +1 ... but I prefer putting the graphs into the third dimension:http://math.stackexchange.com/questions/392/intuitive-understanding-of-the-derivatives-of-sin-x-and-cos-x/1093#1093 . :) – Blue Apr 07 '14 at 01:51
  • 23
    This is the normal way of introducing sine and cosine at high schools. At least in our country. – Vladimir F Героям слава Apr 07 '14 at 15:36
  • Does this also explain why $\sin^2 + \cos^2 = 1$ – Tyler Hilton Apr 07 '14 at 16:02
  • Outstanding animation! Shows how so many of the concepts within trigonometry are interrelated! – Nathaniel Payne Apr 07 '14 at 16:31
  • @TylerHilton I could show you that using a graph. But in all seriousness, if you imagine $sin^2 + cos^2 = 1$ as the Pythagorean theorem, and you use the unit circle (where $sin$ is the $y$ and $cos$ is the $x$), you can figure it out yourself. – Cole Tobin Apr 11 '14 at 01:19
  • 20
    @joeA: I've made a programmatic version of this on Khan Academy; feel free to change the speed to your liking: https://www.khanacademy.org/cs/circle/4597064320155648 – BHSPitMonkey Apr 16 '14 at 10:48
  • I think one understands nothing but will believe everything if one looks at this moving picture. That really is good "1st April" joke. – miracle173 May 14 '14 at 01:15
  • 10
    I'd love to see the hyperbolic version. – ThomasMcLeod Jun 02 '14 at 02:54
  • Probably obvious to some but why do the sine waves more right to left and bottom to top vs the other way around? – cavalcade Sep 22 '14 at 09:47
  • Why community wiki here? You'd have gained a decent amount of upvotes by now :P Not that it's important; just that I like accruing rep for giving bounties.. – Panglossian Oporopolist Mar 11 '15 at 12:29
  • 1
    @MattTagg It is running down from $x=0$ to $x=1,2,3,4,\dots$, thus, it moves from left to right. Since we hold the circle in the center and move the graph, we end up with the wave going backwards, that is, right to left. Parametric equations :) – Simply Beautiful Art Nov 12 '16 at 17:49
  • But now the question is, why is the circle spinning anti-clockwise? ;) – Mr Pie Feb 23 '20 at 16:52
  • As a kid one night I dreamed about such a wooden jig and woke up the next morning finally understanding trigonometry. – Museful Jun 07 '20 at 18:32
  • @user148298: True, but easier said than done, because of the symbol barrier. Here’s the link to a discussion of the symbol barrier: https://brainquake.medium.com/solving-the-ultimate-math-problem-b96597fc3534 –  Nov 18 '20 at 12:11
1040

My favorite: tell someone that $$\sum_{n=1}^{\infty}\frac{1}{2^n}=1$$ and they probably won't believe you. However, show them the below:

enter image description here

and suddenly what had been obscure is now obvious.

eykanal
  • 145
  • 1
  • 1
  • 8
  • 19
    My first intuitive visualization of this sum was a circle. I wasn't $100\%$ sure that the answer was $1$(long time back, I had never seen an infinite series before, and 1 was just my first immediate thought) :) – Guy Mar 31 '14 at 17:38
  • "infinite squares"? – Jack M Apr 01 '14 at 10:43
  • 2
    @JackM - Added some text to make the point more clear. – eykanal Apr 01 '14 at 12:09
  • 319
    I still don't believe you. – Ojonugwa Jude Ochalifu Apr 02 '14 at 08:51
  • 3
    Well it really depends if you see that, $$\frac{1}{2^n}-\frac{1}{2^{n+1}}=\frac{1}{2^{n+1}}$$ it is easy(logically $\frac{1}{2^{n+1}}$ tends 0 as n tends infinity). – kingW3 Apr 03 '14 at 16:13
  • 1
    This one's nice... and it demonstrates a different way of viewing Zeno's paradox. – avgvstvs Apr 03 '14 at 22:29
  • 1
    While it's not quite as stunning a concept, I'm a big fan of the 'in reverse' version of this proof that shows that $\sum_{i=0}^n \left(F_i\right)^2=F_nF_{n+1}$, with $F_n$ the Fibonacci numbers... – Steven Stadnicki Apr 04 '14 at 15:10
  • 159
    Another way to think of this is that 1/2 + 1/4 + 1/8 = 0.111...binary = 0.999...decimal = 1. – Justin L. Apr 07 '14 at 03:02
  • 32
    You don't need to divide the square into such complicated fragments, just use vertical lines (assuming the x coordinate ranges from 0 to 1) at x=1/2, x=1/4, x=1/8 etc. Each time you add 1/2^n to the area, and in the limit you obviously get 1. – Maxim Umansky Apr 07 '14 at 04:05
  • 68
    @MaximUmansky: That way you'd just get lines that get closer together and it'll be not as obvious. Here, you see the fractions $\frac{1}{2}$ and $\frac{1}{4}$ in their "standard shape", so what remains must _obviously_ be $\frac{1}{4}$. Then, put the same shapes inside the remaining square which is _of the same proportions as the initial one_ (and it's easily checked that $\frac{1}{2}\cdot\frac{1}{4}=\frac{1}{8}$); you'll get the next smaller square, hidden deeper in the corner. Repeat, and the square will shrink to a tiny dot (not a whole line, which may intuitively seem larger). – nobody Apr 09 '14 at 20:09
  • 6
    Along the lines of @MaximUmansky's comment, a two dimensional picture seems unnecessarily complicated. You could just draw a line segment, cut it in half, cut one of the remaining pieces in half, etc. – littleO Apr 09 '14 at 22:59
  • 2
    I tried this once on a bunch of children about 10 years old. Part of the exercise was to color the subdivisions with crayon. They had fun coloring in the figure but I later found out they apparently never understood the convergence. – David K Aug 28 '14 at 21:07
  • 2
    Hmm, never occured to me anyone wouldn't believe $\sum 1/2^n = 1$. I always assumed everyone would believe the flea hopping halfway across the remaining distance would add to total distance. – fleablood Apr 09 '16 at 01:29
  • 1
    Another way to think of this is that $$\frac{1}{1-x} = 1 + x + x^2 + x^3...$$ for $-1 – MCCCS Jul 22 '17 at 08:48
  • 6
    Yet another way to think of it is just rewriting 1 infinitely many ways: $$1=1/2+1/2=1/2+1/4+1/4=1/2+1/4+1/8+1/8=1/2+1/4+1/8+1/16+1/16=\cdots$$ and so on. – Arbutus Jun 28 '18 at 02:03
  • Strongly agree with @MaximUmansky. The two dimensional picture is unnecessarily complicated. – Brady Gilg Apr 22 '19 at 16:28
  • It's also very close to me. I ask my brother,"Can you solve this series, 1/2+1/4+1/8+1/16+1/32+1/64+1/128+........?" Then when he says no..I tell him I have cut just one into these pieces. So, if you find the sum of this fractions, you will find 1. – Himel May 30 '20 at 09:09
732

This visualisation of the Fourier Transform was very enlightening for me:

enter image description here

The author, LucasVB, has a whole gallery of similar visualisations at his Wikipedia gallery and his tumblr blog.

LucasVB
  • 399
  • 1
  • 10
njp
  • 189
  • 1
  • 2
  • 7
  • 1
    Can someone please confirm my interpretation of this graphic? From what I understand, you have some $f$ wave which needs to represented by sins and cos'. Then each (blue) curve is a sin/cos representation of $f$ with each successive one becoming more precise (?). $\hat{f}$ is the height (?) of each curve? Is that what a fourier transform is? – Jeel Shah Apr 07 '14 at 01:25
  • 19
    @gekkostate You're a bit off. f is the sum of multiple simple waves, all with different frequencies and phase angles. The fourier transform takes a complex wave from a given time period, and gives you the phase angles and frequencies of all of the component waves. f^ is the amplitude of each component wave. – Joel Apr 07 '14 at 02:15
  • 13
    I'm down-voting this because this animation doesn't include phase information. The phase is the difference between a pulse and cw. If you don't want it, you should use another basis (wavelets). – Mikhail Apr 07 '14 at 03:19
  • @Joel Ahh! Thanks! I get it now `:D`. – Jeel Shah Apr 07 '14 at 06:21
  • 1
    How does this then explain the continuous transform? – Thomas Ahle Apr 07 '14 at 13:00
  • 15
    this is pretty cool too: http://bl.ocks.org/jinroh/7524988 – ilia choly Apr 07 '14 at 20:54
  • 9
    I saw this animation a long time ago, and it helped me finally understand what the Fourier transformation _does_, but I still am lost as to how it _works_... – Cole Tobin Apr 11 '14 at 01:21
  • 2
    @ColeJohnson the [continuous Fourier transform](https://en.wikipedia.org/wiki/Fourier_transform#Definition) is an [inner product](https://en.wikipedia.org/wiki/Hilbert_space#History), which has many of the properties of the dot product in a Euclidean space. In particular, two functions are *orthogonal* if their IP is zero. $\hat{f}(\omega)$ is simply the inner product of $f$ with a wave function with frequency $\omega$. The Fourier transform is the spectral coefficients in an infinite basis of wave functions, just like a vector times a basis matrix is the spectral coefficients in that basis. – Emil Lundberg May 15 '14 at 18:56
  • 23
    I don't find this graphical example helpful in explaining the Fourier Transform. If anything, it is going to add to existing confusion people have with the Fourier Series. The sin and cos expression shown is too general to be meaningful, and the graphic shown for the "transform" is really just a visualization of the magnitudes of the Fourier Series coefficients you would have if the partial sum function were presumed to be infinitely periodic. When I calculate the actual Fourier Transform of the segment of function shown, I get something quite different: http://imgur.com/eQuerJa – Adam Smith Aug 31 '14 at 18:53
  • 1
    IMO fourier transforms are not "easy to explain". – Ruben Jun 20 '19 at 21:38
  • Cut the last term in half and you get a nicer looking approximation. – richard1941 Jan 29 '20 at 03:03
  • 5
    @ColeJohnson, try this link then, I think this 3Blue1Brown video is an even nicer explanation of how Fourier Transform works: https://youtu.be/spUNpyF58BY – Zoltan Mar 02 '20 at 21:03
  • We can say it in simpler terms, @AdamSmith: This illustration is of the **fourier series**, not the transform. The actual transform illustration looks like this instead: https://en.wikipedia.org/wiki/File:Continuous_Fourier_transform_of_rect_and_sinc_functions.gif – creanion Dec 19 '21 at 11:24
679

Here is a classic: the sum of the first $n$ positive odd numbers $= n^2$.

enter image description here


We also see that the sum of the first $n$ positive even numbers $= n(n+1)$ (excluding $0$), by adding a column to the left.

enter image description here

Yiyuan Lee
  • 14,105
  • 4
  • 38
  • 66
  • 32
    If you remove the leftmost column, you get a proof that the sum of the first $n$ non-negative even numbers (counting 0) is $n(n-1)$ – becko Apr 03 '14 at 16:29
  • 36
    I read something about how Galileo noted that the distance a falling body covered over a unit time went as the series of odd numbers and was confused until I realized this. – Nick T Apr 03 '14 at 17:37
  • 4
    I have noticed an interesting method to calculate a square root based on this fact: https://disqus.com/home/discussion/geeksforgeeks/babylonian_method_for_square_root/#comment-2145885386 – sergiol Sep 07 '15 at 23:46
604

The sum of the exterior angles of any convex polygon will always add up to $360^\circ$.

enter image description here

This can be viewed as a zooming out process, as illustrate by the animation below:

enter image description here

Yiyuan Lee
  • 14,105
  • 4
  • 38
  • 66
  • 96
    I always found the easiest way to think about this was that if you move around a convex shape such that you eventually end up at the start, you must have rotated through exactly 360º. If you walk around *any* shape, and count anticlockwise rotations as negative, but clockwise rotations as positive, I imagine the angles will still add to 360º (or -360º depending on which direction you take). – daviewales Apr 05 '14 at 12:27
  • 26
    I like to view this as a limiting process. Imagine that the picture on the right is the picture on the left zoomed out a great distance. – Steven Gubkin Apr 05 '14 at 14:10
  • 8
    This would be better as an animation (which achieves the zooming effect). – Duncan Jones Apr 07 '14 at 10:31
  • 6
    Edited. Do have a look. (It may look horrible, since this is the first time I made an animation.) – Yiyuan Lee Apr 07 '14 at 13:36
  • The animation is worse (I didn't even know what it was supposed to be proving until I looked in the edit history), because it doesn't mark the angles the way your other images did. Maybe if you put both the animation and the other images? – Muhd Apr 11 '14 at 21:41
  • @daviewales Yes, this approach works for any shapes, but you don't always end up with $360^\circ$. For instance, a pentagram comes out at $720^\circ$. – Emil Lundberg May 15 '14 at 19:03
  • @EmilLundberg, I'm not quite sure how you got 720º. I just loaded a picture of a pentagram, then "walked" around it by pointing my thumb in the direction of motion, and when I got back to the start, my hand was still attached to my wrist. Therefore angle of rotation << 720º. – daviewales May 20 '14 at 07:54
  • 1
    @daviewales, then you must have a very flexible wrist. ;) I imagine starting halfway along one of the sides, and walking along the edges. If you're facing south-southwest when you start, by the time you arrive back at the starting position you'll have turned past north twice in the same direction of rotation – Emil Lundberg May 20 '14 at 09:13
  • I still don't know what you mean! – daviewales May 20 '14 at 10:55
  • @daviewales: He means a star shape like ⚝. And by the way it's possible to keep your thumb pointing tangent to a closed curve on a horizontal plane with arbitrary winding number by moving your elbow above or below the plane appropriately.. – user21820 May 26 '14 at 07:08
  • Ant it would be way cooler if it would also go backwards! – scjorge Sep 03 '14 at 14:01
  • the gif reminds me of Aperture Industries – Madeyedexter Dec 07 '15 at 05:50
  • That doesn't explains why convexity is required does it? – Amr Dec 15 '15 at 15:22
  • And this fails on a sphere or the hyperbolic plane, precisely where you can't zoom! ('Cause "similar" is the same as "congruent" on those) – Akiva Weinberger May 20 '16 at 03:20
  • 9
    this is really,really clever – Konstantinos Gaitanas Aug 03 '17 at 09:07
  • This is best done as an activity (I know it was done as an activity in school geometry class). Students could draw on paper and cut exterior angles to see the sum of the angles they make. – Cyriac Antony Jan 03 '19 at 04:32
  • 1
    Not to detract from the wonderful answers, but once one knows of the sum of the interior angles of an n-gon being (n-2)*180 degrees, the missing 2*180 degrees from the n straight angles at the n vertices must be the sum of the external angles. (Or is that traditionally how it's taught? Over a half-century since I was in high school, so apologies...) – Dr. Michael W. Ecker Feb 06 '19 at 21:55
  • If you ever played with Logo Turtle as a kid this is one of life's self-evident axioms. – Museful Dec 28 '20 at 13:17
573

A well-known visual to explain $(a+b)^2 = a^2+2ab+b^2$:

$(a+b)^2 = a^2+2ab+b^2$

Johannes
  • 1,062
  • 1
  • 6
  • 18
  • 10
    Always liked this one – Almo Apr 02 '14 at 14:11
  • 14
    why area of a^2 is greater tham area of b^2? – jaczjill Apr 03 '14 at 07:34
  • 84
    @jaczjill Because a and b are not the same number. – Conor Pender Apr 03 '14 at 11:24
  • 41
    @jaczjill the areas of a and b are arbitrary. They could be any size and it would make no difference. It's just an example. – theyetiman Apr 04 '14 at 08:29
  • 13
    Have you seen the Montessori [binomial cube](http://www.infomontessori.com/sensorial/visual-sense-binomial-cube.htm)? – MJD Apr 04 '14 at 14:30
  • 1
    @jacziill if you take a=b then both will have equal area, if a>b then a^2 will have a greater area and vice versa. – Soham De Mar 12 '17 at 15:47
  • 3
    It occurred to me that, while there's a good way to visualize $(a+b)^2=a^2+2ab+b^2$ for $a,b$ real, and while there's a good way to visualize complex multiplication (as scaling and rotation), there does not seem to be a good way to visualize $(a+b)^2=a^2+2ab+b^2$ for $a,b$ complex. – Akiva Weinberger Jul 27 '17 at 01:29
  • 1
    This is how I teach students I tutor that ^2 isn't linear – Riley Jul 17 '20 at 18:46
440

While attending an Abstract Algebra course I was given the assignment to write out the multiplication table modulo n. I forgot to do the homework until just before class, but it was so easy to write the program I was able to print the result between classes.

The circular patterns in the tables fascinated me, and compelled me to replace the numbers with colors. The result is a beautiful illustration showing the emergence of primes and symmetry of multiplication.

The colors were chosen to start blue at 1 (cold) and fade to red at n (hot). White is used for zero (frozen), because it communicates the most information about prime factorization.

The interactive version can be found here: https://web.archive.org/web/20140830110358/http://arapaho.nsuok.edu/~deckar01/Zvis.html

Multiplication of the integers modulo 15:

enter image description here

Multiplication of the integers modulo 512:

enter image description here

Jared Deckard
  • 101
  • 1
  • 1
  • 4
  • 16
    This is awesome. Looks very much like a fractal. Is there a name for this fractal? – OmnipotentEntity Apr 07 '14 at 15:52
  • What in specific is it lacking? Correct me if I'm wrong, but if we take the white dots to be the area of the fractal and the red and blue to be the border then it's an object of fractional dimension (either 0d points or 1d lines filling a 2d plane) and self-symmetric. – OmnipotentEntity Apr 07 '14 at 17:42
  • 43
    @OmnipotentEntity it doesn't qualify as having fractional dimension because the underlying space is discrete. If you use a real space instead - the set of x, y such that x * y = 0 mod n, you get a set of hyperboles (linear in dimension). If you take a limit towards infinite squares, then the count of white squares is equal to the length of the edge whenever the length is prime. – John Dvorak Apr 07 '14 at 18:48
  • I just read about [qualitative self-similarity](http://en.wikipedia.org/wiki/Fractal#Characteristics) which I think describes the relationship between frames in this example, but it isn't continuous in any dimension. – Jared Deckard Apr 07 '14 at 18:51
  • 3
    I made the exact same picture when I took group my first group theory class. Wonderful. – Seth Apr 07 '14 at 20:24
  • 1
    +1 for nostalgia. I remember programming in QBASIC and using x * y as the index in the color palette to see this pattern emerge. – Ruben Apr 09 '14 at 03:13
  • 7
    Although not colored WolframAlpha also does a nice job of illustrating this: http://www.wolframalpha.com/input/?i=Z_200 – Vikram Saraph Apr 09 '14 at 23:05
  • Interesting, but its gray scale and they don't highlight zeros. Also it stops visualizing operations after n gets too high and just outputs a fairly worthless list of group elements. – Jared Deckard Apr 10 '14 at 00:22
  • I would like to see the shape for higher numbers...wolfram stops plotting too soon. – Integral Apr 10 '14 at 00:24
  • The curves seen are all roughly scaled copies of $xy=1$. – anon Apr 13 '14 at 19:49
  • Can you explain how do you calculate each item? – Pedro77 Aug 27 '14 at 20:55
  • Perhaps it would be better to assign each number to a hue (to communicate 1 and n-1 are rather close.) – PyRulez Jan 17 '15 at 00:02
  • 10
    The 'standing up too fast' fractal – Pigeon Apr 22 '15 at 14:32
  • @pedro, this is a Cayley table, without the rows and columns labeled. Imagine that each row and column are labeled, from left to right and top to bottom, from 1 to n. Then the cooresponding grid 'tile' at row x and column y would be x × y. Except in this case, × is defined as multiplication modulo n. – galois Jan 03 '16 at 07:47
  • 1
    @PyRulez Using your hue idea I made this in Mathematica: http://i.imgur.com/xdaCEfD.png Code was: `ArrayPlot[Table[Table[Mod[m*n, 512], {m, 0, 511}], {n, 0, 511}], ColorFunction -> Hue, ColorFunctionScaling -> True, AspectRatio -> Automatic, Frame -> False, PixelConstrained -> True, ImageSize -> 512]` – Mark S. Apr 09 '16 at 01:46
  • plot the $N \times N$ matrix $M_{ij}(N) = \lfloor \frac{N}{ij} \rfloor$ and remember that the Riemann hypothesis is that $\delta_1^T M(N)^{-1} 1 = \mathcal{O}(N^{1/2+\epsilon})$ (with $1$ the vector with only ones) – reuns Apr 22 '16 at 21:06
  • Looks like a Moire pattern. Is there any way to interpret this in terms of aliasing? – The_Sympathizer Jan 17 '17 at 03:06
  • 1
    @mike4ty4 It could be thought of as an interference pattern produced by the intersection between of a set of concentric hyperbolic curves and a two dimensional grid. Changing the resolution of the grid modulates the set of curves, so isn't quite the same as aliasing. Interestingly, aliasing this pattern (by rendering large tables and downsampling them by averaging pixels) results in a nearly flat purple image. It is in fact the absence of aliasing that produces the pattern. – Jared Deckard Jan 18 '17 at 04:48
  • @Jared Deckard: I meant, could it be interpreted as an aliased version of a pattern with higher resolution? What do the hyperbolic curves represent? – The_Sympathizer Jan 18 '17 at 04:50
  • This feels like one of those "can you see the sailboat" optical illusions. I wish I could split on the y-axis and then horizontally reverse one of the sides and look at it stereoscopically. – EngrStudent Aug 26 '20 at 15:24
  • Wait, you mean they force students to calc a table like that by hand? Not a Russian math school, definitely. – Dzmitry Lazerka Sep 11 '21 at 06:40
  • This can be interpreted as DFT matrix. By permeating rows and columns of this matrix, this matrix is transformed into a group of small DFT matrices. This is a core of many FFT algorithms. https://en.wikipedia.org/wiki/Rader%27s_FFT_algorithm – Hayashi Yoshiaki Nov 20 '21 at 22:45
400

Simple answer for "what is a radian":

Logarithmic spiral and scale:

Gallifreyan
  • 747
  • 6
  • 11
user2804123
  • 51
  • 1
  • 1
  • 2
366

When I understood Fourier series visually-

Fourier series of square wave

Martin Sleziak
  • 50,316
  • 18
  • 169
  • 342
Gopi
  • 101
  • 1
  • 1
  • 3
  • 19
    OH. Wow. I just took my signals course and even after I did well on the exam I still didn't actually UNDERSTAND fourier series. Thanks, wow. – VicVu Apr 02 '14 at 22:25
  • 51
    This diagram is a particularly good illustration of the Gibbs Phenomenon, too. Nice! – Steven Stadnicki Apr 04 '14 at 06:09
  • 11
    I don't understand what this is showing: is it showing hte approximation of a square wave by a Fourier series? – ShreevatsaR Apr 05 '14 at 05:18
  • 7
    @ShreevatsaR Yes. It's an illustration of the Gibbs Phenomenon. – Potato Apr 05 '14 at 06:01
  • @ShreevatsaR: I evaluate the limiting amount of overshoot in the Gibbs Phenomenon in [this answer](http://math.stackexchange.com/a/698775) (about $8.9\%$ of the size of the discontinuity). – robjohn Apr 06 '14 at 12:03
  • 2
    I like this one more to show the difference between pointwise and uniform convergence: this Fourier series only converges pointwise and not uniformly. The total result will always be off (known as Gibbs phenomenom): this wouldn't happen if there was uniform convergence. – Ruben Apr 09 '14 at 03:09
  • This makes me wonder if there's an extension of $f(x) = \sum_{k=1}^n \frac{sin(kx)}{k}$ to real values of $n$. – Dolda2000 Sep 03 '14 at 02:36
  • @Dolda2000 yes, but there's not a unique one. One idea would be the linear interpolation $\sum_{k=1}^{n+r}\sin(kx)/k=r\sin[(n+1)x]/(n+1)+\sum_{k=1}^n \sin(kx)/k$, where $0\leq r<1$. – Kevin Arlin Sep 09 '14 at 23:05
  • best way of understanding fourier! – pooja somani May 19 '17 at 08:43
  • Only one more upvote needed to get you on 300! (And I've run out now) – Mr Pie Feb 23 '20 at 16:56
346

enter image description here

Here is a very insightful waterproof demonstration of the Pythagorean theorem. Also there is a video about this.

It can be explained as follows. We seek a definition of distance from any point in $\mathbb{R}^2$ to $\mathbb{R}^2$, a function from $(\mathbb{R}^2)^2$ to $\mathbb{R}$ that satisfies the following properties.

  • For any points $(x, y)$ and $(z, w)$, $d((x, y), (x + z, y + w)) = d((0, 0), (z, w))$
  • For any point $(x, y)$, $d((0, 0), (x, y))$ is nonnegative
  • For any nonnegative real number $x$, $d((0, 0), (x, 0)) = x$
  • For any point $(x, y)$, $d((0, 0), (x, -y)) = d((0, 0), (x, y))$
  • For any points $(x, y)$ and $(z, w)$, $d((0, 0), (xz - yw, xw + yz)) = d((0, 0), (x, y))d((0, 0), (z, w))$

Suppose a function $d$ from $(\mathbb{R}^2)^2$ to $\mathbb{R}$ satisfies those conditions, then for any point $(x, y)$, $d((0, 0), (x, y))^2 = d((0, 0), (x, y))d((0, 0), (x, y)) = d((0, 0), (x, y))d((0, 0), (x, -y)) = d((0, 0), (x^2 + y^2, 0)) = x^2 + y^2$ so $d((0, 0), (x, y)) = \sqrt{x^2 + y^2}$ so for any points $(x, y)$ and $(z, w)$, $d((x, y), (z, w)) = \sqrt{(z - x)^2 + (w - y)^2}$ Now I will show that $d((x, y), (z, w)) = \sqrt{(z - x)^2 + (w - y)^2}$ actually satisfies those properties. It's trivial to show that it satisfies the first 4 conditions. It also satisfies the fifth condition because for any points $(x, y)$ and $(z, w)$, $d((0, 0), (xz - yw, xw + yz)) = \sqrt{(xz - yw)^2 + (xw + yz)^2} = \sqrt{x^2z^2 - 2xyzw + y^2w^2 + x^2w^2 + 2xyzw + y^2z^2} = \sqrt{x^2z^2 + x^2w^2 + y^2z^2 + y^2w^2} = \sqrt{(x^2 + y^2)(z^2 + w^2)} = \sqrt{x^2 + y^2}\sqrt{z^2 + w^2} = d((0, 0), (x, y))d((0, 0), (z, w))$

As a result of this, from now on, I will define the distance from any point $(x, y)$ to any point $(z, w)$ as $\sqrt{(z - x)^2 + (w - y)^2}$ and denote it as $d((x, y), (z, w))$. I will also use $d(x, y)$ as shorthand for $d((0, 0), (x, y))$ Since distance satisfies condition 5, for any right angle triangle at all, not just ones whose legs are parallel to the axes, the square of the length of its hypotenuse is equal to the sum of the squares of the lengths of its legs.

enter image description here

This image shows that using that definition of distance, for any right angle triangle whose legs are parallel to the axes and have lengths $x \in \mathbb{R}^+$ and $y \in \mathbb{R}^+$, the area of a square with the hypotenuse as one of its edges is $(x - y)^2 + 2xy = x^2 + y^2 = (d(x, y))^2$. Combining that result with the fact that distance satisfies condition 5, we can show that for any right angle triangle, even with legs non parallel to the axes, the area of a square with its hypotenuse as its edge has an area equal to the sum of the squares of the lengths of its legs.

enter image description here

Sources:

Timothy
  • 741
  • 7
  • 16
Maxim Umansky
  • 449
  • 1
  • 3
  • 14
  • 108
    This does not actually demonstrate the pythagorean theorem. It is showing that for one single right triangle, the theorem holds. This could be a coincidence. Someone seeing this might think that 3,4,5 is the only pythagorean triple. – Sparr Apr 07 '14 at 16:06
  • @Sparr is it not? – Cole Tobin May 15 '14 at 23:17
  • 3
    It looks cool, but provides very little (no?) insight about why PT is true. – Cheerful Parsnip Aug 29 '14 at 01:25
  • 9
    @GrumpyParsnip Before you learn why something is true, you might want to do a sanity check to verify that it probably is. This does that, while looking cool. The only thing that I really care about is that it looks cool, though. Mathematics is a very aesthetic thing, so I'm allowed to like something just because it looks nice. – Samuel Yusim Sep 05 '14 at 00:06
  • I saw this example in a science museum when I was in high school back in the early 90's. I have always remembered it. – Fezter Nov 29 '16 at 06:08
  • actually did this in middle school. – pooja somani May 19 '17 at 08:45
  • This looks nice but actually did't show anything about the Phytagorean Theorem. You can construct a machine where at the hypotenusis is mounted a rectangle with a site perpendicular to the hypothenusis is half as long as the hypothenusis. So the area of the rectangle is half the area of the square in this video. But the machine shows the same behaviour. The liquid of the squares fill exactly the hypothenusis square. This is because the hypothenusis box is two times as thick as the other two square boxes. It only describes what the PT claims. – miracle173 May 07 '18 at 19:58
  • Make a computer animation where arbitrary areas mounted at the catheti fill an area mounted on the hypothenus. It is simple to convince the eye even it is mathematically completely wrong. So thats no proof. – miracle173 May 07 '18 at 19:59
  • The water demonstration does not serve as a proof - it is a visualisation - it conveys what the dry pythagorean theorem actually can mean in the real world. – creanion Dec 19 '21 at 11:28
  • @miracle173 I'm not a fan of this liquid demonstration either, but just note that the use of translucent water against a white background serves as a bit of a check against that kind of fakery. – Kevin Bourrillion May 18 '22 at 14:50
  • @KevinBourrillion But even if I believe that that what I see is true. I cannot see it the sum of the water of the two squares is exactly the sum of the hypothenuses square. I also cannot see if this will work for other rectangular triangles – miracle173 May 18 '22 at 15:12
  • Yes, these are some of the reasons I hate this gif too. – Kevin Bourrillion May 19 '22 at 14:59
312

Subdividing circle

This is a neat little proof that the area of a circle is $\pi r^2$, which I was first taught aged about 12 and it has stuck with me ever since. The circle is subdivided into equal pieces, then rearranged. As the number of pieces gets larger, the resulting shape gets closer and closer to a rectangle. It is obvious that the short side of this rectangle has length $r$, and a little thought will show that the two long sides each have a length half the circumference, or $\pi r$, giving an area for the rectangle of $\pi r^2$.

This can also be done physically by taking a paper circle and actually cutting it up and rearranging the pieces. This exercise also offers some introduction to (infinite) sequences.

Ben Rowland
  • 1
  • 1
  • 1
  • 2
  • 1
    assuming the student knows about pi, this is awesome! – Filip Haglund Apr 07 '14 at 20:52
  • 36
    This doesn't work for me :( Sadly, I find it non-obvious that the length of the bottom really will converge helpfully to $\pi r$. – Nicholas Wilson Apr 08 '14 at 16:50
  • 20
    @NicholasWilson By definition, the circumference of a circle is π times the diameter of the circle. Here, we've cut the circle into slices, half flipped one way, half flipped the other. Therefore half the circumference (πd) is on the bottom. Half of the diameter is the radius, *r*. Does that help? – ghoppe Apr 08 '14 at 19:29
  • 60
    @ghoppe Nice try! But it's all wiggly. We're lucky that $\sin{\theta} \sim \theta$ as $\theta \rightarrow 0$, so the actual perimeter of the bottom (which is known to be $\pi$) does converge to the width of the rectangle. But - that nice property of $\sin$ is exactly what we're trying to prove! – Nicholas Wilson Apr 09 '14 at 08:20
  • 1
    @NicholasWilson Yes, it's wiggly! But we can visually infer from the diagram that each time we slice, it gets less wiggly. If you could slice infinite slices, it will be straight! It's a similar process to inscribing polygons of increasing sides on the inside and outside of the circle to approximate the circumference. – ghoppe Apr 09 '14 at 16:29
  • 12
    @ghoppe What would you visually infer about the perimeter of a Koch snowflake? I've had people assure me "It must be bounded!" Visual inferences are susceptible to error :( You need to invoke some limit arguments to convince me those arcs on the segments do converge. Similarly, inscribing polygons to determine the _circumference_ sounds very dangerous (think what would happen if you tried that with a Koch snowflake) -- but bounding the _area_ above and below by inscribed/exscribed polygons is definitely a sound proof. – Nicholas Wilson Apr 09 '14 at 17:56
  • 2
    Archimedes was a clever man, and his inscribed polygons were for determining the _area_. For the same reason, his argument about the area of the sphere has to be very ingenious - not as straightforward as a simple polar integral! – Nicholas Wilson Apr 09 '14 at 18:00
  • 13
    @NicholasWilson coming back to this discussion rather late, but perhaps I can convince you. The length of the bottom is always πr, it doesn't change with iteration so all we have to show is that the shape ends up as a rectangle. This is equivalent to stating that the line between two points on a circle tends to the tangent as the two points get closer together, which I believe is at least one definition of the tangent. – Ben Rowland Dec 16 '14 at 22:16
  • 7
    @BenRowland Nicholas's point is that this type of argument is actually quite subtle and demands more care than just saying the shape "approaches" a rectangle. Consider the famous example of a unit staircase of slope $1$ divided into $n$ steps. For every $n$ the arc length of the staircase is exactly $2$. But as $n\to\infty$ the staircase approaches a diagonal line whose length is quite different from $2$. – Erick Wong Jun 26 '15 at 07:25
  • 3
    THANK YOU!! I asked my non-math-enthusiastic wife a few years ago "why is the pi which is used to calculate the circumference the same as the one that calculates the area... isn't that strange?". After a shrug and eye roll by her, the conversation was over. This has been sitting in the back of my mind for a while and this just made my day. – Frank Bryce Aug 04 '15 at 19:00
  • This is how I proved the area of circle to be $ \pi r^2 $ (obviously after telling the integration one) in my job interview ^_^ – ABcDexter Jan 05 '16 at 18:08
  • 1
    @NicholasWilson "We're lucky that $\sin \theta \sim \theta$ as $\theta \to 0$ [...] But - that nice property of $\sin$ is exactly what we're trying to prove!" This is wrong in an important way that makes the argument sound vacuous when it is not! This property relates to our *definition* of $\pi$ (in terms of the circumference) and is being used to establish the *new result* that the area is related to $\pi$. The only hidden assumption being used is of course that the circle is smooth, which is of course necessary. – not all wrong Feb 01 '18 at 17:46
  • That's the way I learned area of circle when I was grade 5 in school and I still remember it. – MonkeyKing Aug 03 '18 at 05:25
  • @NicholasWilson: Sorry but I agree with ghoppe: it is evident that the bottom becomes a straight line at $\infty$; one can make this precise with uniform convergence. With an elementary reasoning about angles, we can see that the maximum of the boundary function approaches to zero. This is traslating into formality what is evident from the drawing :) – Andrea Marino Jun 29 '19 at 17:12
  • 2
    I think it misses some basic statement. The statement should be "This demonstrates that the area is $\pi r^2$ **given** that the circumfence is $2r\pi$." (or alternatively the vice versa argument) – Raphael J.F. Berger Nov 20 '19 at 13:26
292

A visual explanation of a Taylor series:

$f(0)+\frac {f'(0)}{1!} x+ \frac{f''(0)}{2!} x^2+\frac{f^{(3)}(0)}{3!}x^3+ \cdots$

or

$f(a)+\frac {f'(a)}{1!} (x-a)+ \frac{f''(a)}{2!} (x-a)^2+\frac{f^{(3)}(a)}{3!}(x-a)^3+ \cdots$

Taylor series gif

When you think about it, it's quite beautiful that as you add each term it wraps around the curve.

Peter Mortensen
  • 627
  • 5
  • 11
joshlk
  • 275
  • 1
  • 3
  • 13
  • 25
    It is interesting to note that Taylor series are relatively very precise near their center, which makes them especially useful in computer science when you need a precise but still fast approximation. For example, processors do not have a `cosine` function built-in, but will fly through an equation such as `x => 1 - (x^2)/(2!) + (x^4)/(4!) - ...` and from this equation, you could build a function that accepts a `precision` parameter so that the call recurses until higher accuracy is achieved. (note: there are many more efficient ways to do this) – sleblanc Apr 03 '14 at 21:55
  • 1
    @sebleblanc It is just amazing that petty much any function can represented with a Taylor series locally – joshlk Apr 04 '14 at 08:15
  • 5
    @sebleblanc Should have said "[_some_] processors" instead. x86 processors have had a cosine function [for some time now](http://en.wikipedia.org/wiki/X87) with the [FCOS](http://en.wikipedia.org/wiki/X86_instruction_listings#Added_with_80387) instruction. It doesn't do it with a taylor series as far as I can tell. What I assume it does is use a lookup table followed by some sum/difference formulas. – Cole Tobin Apr 04 '14 at 16:05
  • You are right that X87 co-processors are omni-present, but there are far more embedded CPUs in the world than there are X86/87 CPUs. Just think of how many devices you interact with that are not strictly computers. ///// From my desk, I can see three computer monitors, one TV, two video game consoles, one oscilloscope, one modern refrigerator, two cordless phones, a wireless mouse, a microcontroller dev board, a Raspberry Pi, my cell phone, my *credit card*. All of these have a processor that does not have the `FSIN` instruction... If anything, I should have said [most] instead. ;-) – sleblanc Apr 12 '14 at 14:31
  • 5
    @sebleblanc The x86/87 FPU uses [CORDIC](http://en.wikipedia.org/wiki/CORDIC) which lets you compute trigonometric functions in hardware using only bit shifts and addition. – Calmarius Aug 27 '14 at 16:57
286

When I look up "area of a rhombus" on Google images, I find plenty of disappointing images like this one:

enter image description here

which show the formula, but fail to show why the formula works. That's why I really appreciate this image instead:

Jim Wilson University of Georgia

which, with a little bit of careful thought, illustrates why the product of the diagonals equals twice the area of the rhombus.

EDIT: Some have mentioned in comments that that second diagram is more complicated than it needs to be. Something like this would work as well:

enter image description here

My main objective is to offer students something that encourages them to think about why a formula works, not just what numbers to plug into an equation to get an answer.


As a side note, the following story is not exactly "visually stunning," but it put an indelible imprint on my mind, and affected the way I teach today. A very gifted Jr. High math teacher was teaching us about volume. I suppose just every about school system has a place in the curriculum where students are required learn how to calculate the volume of a pyramid. Sadly, most teachers probably accomplish this by simply writing the formula on the board, and assigning a few plug-and-chug homework problems.

enter image description here

No wonder that, when I ask my college students if they can tell me the formula for the volume of a pyramid, fewer than 5% can.

Instead, building upon lessons from earlier that week, our math teacher began the lesson by saying:

We've learned how to calculate the volume of a prism: we simply multiply the area of the base times the height. That's easy. But what if we don't have a prism? What if we have a pyramid?

At this point, she rummaged through her box of math props, and pulled out a clear plastic cube, and a clear plastic pyramid. She continued by putting the pyramid atop the cube, and then dropping the pyramid, point-side down inside the cube:
enter image description here

She continued:

These have the same base, and they are the same height. How many of these pyramids do you suppose would fit in this cube? Two? Two-and-a-half? Three?

Then she picked one student from the front row, and instructed him to walk them down the hallway:

Go down to the water fountain, and fill this pyramid up with water, and tell us how many it takes to fill up the cube.

The class sat in silence for about a full minute or so until he walked back in the room. She asked him to give his report.

"Three," he said.

She pressed him, giving him a hard look. "Exactly three?"

"Exactly three," he affirmed.

Then, she looked around the room:

"Who here can tell me the formula I use to get the volume of a pyramid?" she asked.

One girl raised her hand: "One-third the base times the height?"

I've never forgotten that formula, because, instead of having it told to us, we were asked to derive it. Not only have I remembered the formula, but I can also even tell you the name of the boy who went to the water fountain, and the girl who told us all the formula (David and Jill).


Given the upvoted comment, If high school math just used a fraction of the resources here, we'd have way more mathematicians, I hope you don't mind me sharing this story here. Powerful visuals can happen even in the imagination. I never got to see that cube filling up with water, but everything else in the story I vividly remember.

Incidentally, this same teacher introduced us to the concept of pi by asking us to find something circular in our house (“like a plate or a coffee can”), measuring the circumference and the diameter, and dividing the one number by the other. I can still see her studying the data on the chalkboard the next day – all 20 or so numbers just a smidgeon over 3 – marveling how, even though we all probably measured differently-sized circles, the answers were coming out remarkably similar, “as if maybe that ratio is some kind of constant or something...”

J.R.
  • 171
  • 1
  • 3
  • 10
  • 43
    ^^1 million upvotes for the stories about maths education. I am writing both of these down for future reference. In fact, I think we should start a website with instructions to help primary teachers teach mathematical concepts in interesting ways. – daviewales Apr 05 '14 at 12:34
  • 29
    @daviewales It's been started - the mathematics educators SE. –  Apr 05 '14 at 14:45
  • Huzzah! Now to promote it such that primary teachers are aware of it! – daviewales Apr 05 '14 at 15:10
  • (Also, I was thinking more of a structured site, with ordered categories, etc.) – daviewales Apr 05 '14 at 15:11
  • 9
    Great stories. I do, however, think your first rhombus is fine. I actually find it easier to understand the result using that image than the second. Either diagonal splits the rhombus into two triangles, each of area $$\frac{1}{2}\frac{d_1}{2}d_2=\frac{1}{2}d_1\frac{d_2}{2}.$$ (Is there some aspect-ratio issue with your image? The white rectangle should be a square, but doesn't look like it on my screen.) – Will Orrick Apr 06 '14 at 02:36
  • @WillOrrick - You make a good point. Yet I'm still disappointed in the first image because it doesn't really nudge the learner toward thinking about the area of a rhombus as the area of two adjacent triangles with related dimensions. It just shows the formula; a two-color rhombus might have worked better. (As for the second image, I agree, it doesn't look as square as it should.) – J.R. Apr 07 '14 at 01:39
  • 26
    The first rhombus picture is fine. Just draw a rectangle around it, and it's plain to see that the four triangles that form the rhombus cover half of the rectangle. In contrast, the second picture is so confusing that you yourself got it wrong: you write that it "illustrates why the product of the diagonals equals half of the area of the rhombus", while the ratio is the opposite. – LaC Apr 07 '14 at 03:47
  • Great answer. You should check out [this question](http://math.stackexchange.com/questions/323334/)! – Will Apr 07 '14 at 07:12
  • 1
    @LaC That's what I thought as well. To be honest, I still don't get how the square of side length $(a+b)$ is supposed to explain anything. – Christoph Apr 07 '14 at 07:38
  • @LaC - I'd think the first picture would be fine, too, if it had a rectangle drawn around it, so that it's plain to see that the four triangles that form the rhombus cover half the rectangle. My beef with the first drawing is that most middle schoolers would not think to do that; I'm afraid they'd simply follow the formula without any thought about why it works. – J.R. Apr 07 '14 at 10:23
  • 3
    I like the pyramid story! There is a more geometrical demonstration that 3 pyramids make a cube:http://www.lapasserelle.com/online_courses/maths6/3_pyramids_in_a_cube/index.html And by slicing pyramids with many cuts parallel to the base plane, it can be shifted, much like a rectangular solid loaf of bread can be shifted to approximate a parallelepiped. The more slices, the better the approximation. This slicing and shifting can also make a parallegram into a rectangle or any triangle into half a rectangle. – HopDavid Apr 07 '14 at 15:22
  • HopDavid: YES! That! The water experiment could be a coincidence - if it was 2.855454, you wouldn't know. The cube from 3 pyramids clearly shows it's exactly 3. – Tomáš Kafka Apr 07 '14 at 16:34
  • @HopDavid, you should post that as an answer! – LaC Apr 07 '14 at 18:04
  • @LaC Thanks! But I haven't done any drawings of this notion that I know how to post here. I've done flash animations but don't know how to share flash animations here. For example: http://clowder.net/hop/areas/pararea.html I have a mental image of a demonstration that a pyramid's volume is bh/3, but haven't had the time or energy to make an animated gif or Flash movie. – HopDavid Apr 07 '14 at 20:05
  • I apparently have enough karma to edit so I changed where OP said "half the area of the rhombus" to "twice the area" which is I believe what OP meant. – eeeeaaii Apr 08 '14 at 11:33
  • Your second diagram seems to make the issue much more complicated than simply observing that a diagonal divides the rhombus into two identical triangles. Their base is one diagonal, and their height is half the other diagonal. $$A_R=2A_T=2\left(\frac12 bh\right)=2\left(\frac12 d_1 \left(\frac12 d_2\right)\right)=\frac12 d_1 d_2$$ – Travis Bemrose Apr 13 '14 at 18:23
  • @Travis - Yes, a few folks have made that observation. Perhaps I didn't choose the best examples. I'll stand by my point, though: diagrams showing little more than a polygon, some labels, and an equation often lead a student toward a plug-and-chug mindset that isn't as instructive as it could be. I still think first diagram has plenty of room for improvement; it could be drawn in a way that would do a better job of nudging a student toward your line of reasoning. – J.R. Apr 13 '14 at 18:36
  • @J.R. I understood what you meant at the very beginning by "disappointing", and I heartily agree. However, in the example you chose, I think neither image "points the way" but in the first image it's easier to "find the way". I still don't understand what the second image is getting at. (I'm sure the distortion isn't helping me. What should be a white square of area $s^2$, visibly isn't.) – Travis Bemrose Apr 13 '14 at 20:24
  • @Travis - That's why I gave up and constructed my own image. (Hopefully one that illustrates my point a little better.) I left the other image in my answer so as not to render all these comments obsolete. – J.R. Apr 13 '14 at 20:28
  • I don't get how the pyramid example works. Obviously it shows that that particular prism is three times the area of that particular pyramid, but it doesn't seem to give any feeling as to *why* this is true, or why that particular formula would work for another pyramid. – Daniel McLaury Aug 28 '14 at 00:21
  • 1
    @DanielM - You're right, the experiment isn't a proof. Then again, neither is the formula. I just wanted to draw attention to how my math teacher's technique left a lasting impression that the formula by itself could never manage to do. – J.R. Aug 28 '14 at 01:45
  • 3
    Dave and Jill went down the hall / To fill a cube up with water / A pyramid required three trips / And thus they never forgot 'er. – Blazemonger Jan 08 '15 at 19:49
  • 2
    You didn't derive the formula for the volume of a pyramid. You verified experimentally that it holds. I daresay you still don't know why it holds. You have been taught a formula you could look up in two seconds on the internet. No maths was involved in learning the formula, it was largely a waste of time. If, however, your teacher had actually derived the formula (not too hard) you would have learned an interesting mathematical technique and actually know why the formula works. Your teacher dumbed this down so far that there was no maths left at all. – Peter Webb Feb 22 '15 at 07:31
  • 7
    @Peter - I'll admit that I'm using the word _derive_ a bit loosely for a mathematics forum (it's a soft answer to a soft question), but it's too bad you see this as "dumbing down" and "a waste of time." Most 7th-grade teachers charged with teaching volume would simply write the formula on the board, and it would be well-forgotten by the end of summer. Her technique might have been weak insofar as mathematical rigor goes, but the pedagogy was very strong. I assure you, this woman was not one to "dumb down" anything; I remember re-learning concepts in 11th-grade that she taught us in Jr High. – J.R. Feb 22 '15 at 09:44
  • I'm not sure about others but to me, the formula for the volume of a pyramid is obviously $\frac{1}{3} b \times h$ because the integral of $x^2$ is $\frac{1}{3}x^3$. – Timothy Jan 10 '19 at 00:52
  • I'm not sure that's really the best way to teach. Somebody derived the general formula only after somebody saw how many pyramids of water fill the cube. I read a Stack Exchange answer about a gifted person who taught themself Calculus in school. Maybe it would be better to teach students how to get really good at figuring things out and derive the formula by integration without first seeing how many pyramids of water it takes to fill the cube or being taught the formula. – Timothy Jan 10 '19 at 00:55
  • You can do the volume of a pyramid in much the same way as you've done the rhombus: take two copies of that cuboid you drew, one above the other, giving an $a \times b \times 2h$ cuboid. Cut it into six pieces with planes through each pair of opposite planes. Each of those pieces is a pyramid, and multiplying the area of the base by the height gives $abh$ in all cases. Thus, if you're willing to assume that the volume of a pyramid scales linearly in each dimension (this is easy enough to prove for general polyhedra, if not), they must all have the same volume $2abh/6 = abh/3$. – user3482749 Nov 21 '20 at 20:28
261

As I was in school, a supply teacher brought a scale to lesson:

Source: Wikipedia

He gave us several weights that were labeled and about 4 weights without labels (let's call them $A, B, C, D$). Then he told us we should find out the weight of the unlabeled weights. $A$ was very easy as there was a weight $E$ with weight($A$) = weight($E$). I think at least two of them had the same weight and we could only get them into balance with a combination of the labeled weights. The last one was harder. We had to put a labeled weight on the side of the last one to get the weight.

Then he told us how this can be solved on paper without having the weights. So he introduced us to the concept of equations. That was a truly amazing day. Such an important concept explained with a neat way.

RegDwight
  • 145
  • 1
  • 2
  • 10
Martin Thoma
  • 9,359
  • 14
  • 56
  • 123
  • I do the same thing... except with drawings instead of an actual scale and weights. I need to get this set. – Guillermo Garza Apr 07 '14 at 00:34
  • 22
    @MeemoGarza although drawings are nice, too, I think that an actual device has some advantages: Pupils rather get excited by a real device than by a drawing; it's easier to accept something that you can touch and play with where you instantly get feedback if what you did was correct; as soon as pupils got familiar with the concept of equations, you can explain the physics behind scales ($a_1 \cdot m_1 = a_2 \cdot m_2$ where $a$ is the length of the part of the scale and $m$ is the mass on that side). That way students can see that math is also about describing the real world in a formal way. – Martin Thoma Apr 07 '14 at 05:58
  • 3
    I agree completely, but I don't have the apparatus. I'll look into purchasing one before the next semester. Thanks for helping me make up my mind! – Guillermo Garza Apr 08 '14 at 00:50
  • @MeemoGarza Meanwhile you might be interested in this: http://www.walter-fendt.de/ph14d/hebel.htm - it's a site created by a physics teacher I had in school. This website contains lots of Java applets that show things in physics. – Martin Thoma Apr 08 '14 at 01:59
231

How about a line integral of a scalar field by http://1ucasvb.tumblr.com:

Line integral of a scalar field

Jure Triglav
  • 101
  • 1
  • 1
  • 3
226

This is similar to Aky's answer, but includes a second drawing (and no math.)

To me the second drawing is key to understanding why the $\mathrm c^2$ area is equal to the sum of $\mathrm a^2+\mathrm b^2$.

enter image description here


Edit: comments requested an animation, so a simple gif is attached... enter image description here

Joffan
  • 38,461
  • 5
  • 42
  • 78
Tobia
  • 138
  • 1
  • 1
  • 6
  • 14
    I've always felt this one worked better in [.gif format](http://en.wikipedia.org/wiki/File:Pythagoras-2a.gif). – Patrick M Apr 03 '14 at 20:47
  • 3
    This would be amazing if you animated the change from the first to the second by having the triangles on the right flop over onto the ones on the left to create those rectangles. – Nick Retallack Apr 07 '14 at 00:31
  • 3
    I think this one is lacking because it is not explained in the left picture that the blue and red areas are the same size. – Sparr Apr 07 '14 at 15:58
  • 3
    @Sparr they are not. The salmon area is just 4 times the size of the triangle. The only equivalence is bewteen the blue area on the left (c²) and the blue area on the right (a² + b²) which is just Pythagoras' formula. I will upload a better picture to show it. – Tobia Apr 07 '14 at 16:33
  • Anyway...THIS was my favourite, so, good that you already did it! – MattAllegro May 12 '14 at 21:49
  • Added an animation as suggested – Joffan Jun 09 '16 at 19:50
211

Similarly to eykanal's answer, although demonstrating some interesting facts about medians and geometry as well. It demonstrates that $\displaystyle\sum_{n = 1}^{\infty}\frac{1}{2^n} = 1$: Geometric diagram of triangles

  • 17
    I think this one better. In the square one you have to make a choice between dividing horizontally or vertically. With the triangle there is only one dividing cut. – abnry Apr 05 '14 at 13:55
  • Just nitpicking, but you got the areas of the triangles wrong. For example, the one of are $\frac{1}{2}$ according to you has actually an area of $\frac{1}{2}\times\frac{\sqrt{2}}{2}\times\frac{\sqrt{2}}{2}=\frac{1}{4}$, so what this figure is proving is that $\sum_{n=2}^{\infty}\frac{1}{2^n}=\frac{1}{2}$. – S4M Apr 07 '14 at 07:12
  • 3
    @S4M: The area of the first triangle is *defined* as $1/2$; how is the calculation wrong when there is no calculation... only definition? – Andrew Coonce Apr 07 '14 at 16:51
  • @S4M: Each new triangle bisects the previous one. The full triangle has area 1. The $\frac{1}{4}$ triangle bisects the triangle of area $\frac{1}{2}$ that is left after the $\frac{1}{2}$ triangle and so on. I'm not trying to prove that the right side of the largest bisection line equals the other side (the summation you have), but that the entire collection of triangles approaches 1. –  Apr 08 '14 at 21:17
  • 1
    @AndrewCoonce I was wrong in my previous comment. I asscumend that the side of the main triangle was $1$. It's all fine if its side is $\frac{\sqrt{2}}{2}$. – S4M Apr 09 '14 at 12:27
  • 1
    @S4M do you mean just rad two? – Wildcard Dec 09 '16 at 10:32
184

Steven Wittens presents quite a few math concepts in his talk Making things with math. His slides can be found from his own website.

For example, Bézier curves visually:

Linear interpolation

Order-4 Bezier curve

He has also created MathBox.js which powers his amazing visualisations in the slides.

Christian Chapman
  • 4,427
  • 2
  • 22
  • 43
Kimmo
  • 101
  • 1
  • 1
  • 2
183

Fractal art. Here's an example: "Mandelbrot Island".

An image of "Mandelbrot island".

The real island of Sark in the (English) Channel Islands looks astonishingly like Mandelbrot island: An image of Sark.

Now that I think about it, fractals in general are quite beautiful. Here's a close-up of the Mandelbrot set:

An image of the Mandelbrot set.

user134824
  • 11,504
  • 1
  • 23
  • 47
  • 44
    Are complex numbers easy to explain? – adam.r Mar 31 '14 at 17:15
  • 10
    Probably not. But the idea that an object "looks the same if you zoom in on it" *is* easy to explain, I think. – user134824 Mar 31 '14 at 17:16
  • 1
    @adam.r Sure it is. That's the beauty of it, infinite complexity from simplicity. The Mandelbrot set is created by the iteration of the very simple equation z=z^2 + c. – johnc Apr 01 '14 at 02:34
  • I am not sure whether to agree, or disagree, with @adam.r I mean on one hand, I don't see myself explaining complex numbers to a 5 year old, but the simplicity within the complexity of the Mandelbrot set is just compelling. – Guy Apr 01 '14 at 04:41
  • 9
    Whereas complex numbers might not be easy to explain, it is easy to explain the idea idea that each pixel is calculated by repeating a simple calculation on two numbers (which are initially the coordinates of the pixel), and the color is a measure of how the resulting pair of numbers "escapes" (grows large). – Kaz Apr 01 '14 at 04:57
  • 5
    @adam.r I don't recall precisely when I learned about complex numbers in school, but it was around the 9th grade (maybe 8th, maybe 10th). To understand the Mandelbrot set, nothing more is needed than basic arithmetic with complex numbers. That should put it within reach for anyone who graduated high school. It's certainly a good candidate for this list. – Szabolcs Apr 01 '14 at 16:12
  • This is a close up? I always thought Mandelbrot sets looked the same practically irrespective to zoom. Come to think of it, isn't that part of the definition? – FraserOfSmeg Apr 02 '14 at 22:40
  • 3
    It is a close up. The Mandelbrot set is not completely self-similar, but there are some self-similarities as you zoom. See [this zoom sequence](http://en.wikipedia.org/wiki/Mandelbrot_set#Image_gallery_of_a_zoom_sequence) for an example. – user134824 Apr 03 '14 at 03:12
  • I first tried learning about fractals in 6th grade and didn't understand a _thing_. I learned about complex numbers in 9th grade Trigonometry, so I went back and tried to learn about fractals again. I still couldn't understand what, exactly, is happening. Now I'm almost done with 11th grade and am still fascinated by them and how their simple concepts elude me. – Cole Tobin Apr 04 '14 at 16:02
  • @ColeJohnson, you might want to check this out: http://acko.net/blog/how-to-fold-a-julia-fractal/ – OmnipotentEntity Apr 07 '14 at 19:31
  • 2
    So... how expensive would each foot of shoreline on that island cost... \$0 or \$Infinity. – Brandon Wittwer Aug 27 '14 at 02:56
  • @ColeJohnson, here's a couple more ways of seeing why the Mandelbrot set is a fractal: https://www.quora.com/Why-is-the-Mandelbrot-set-a-fractal – Daniel McLaury Aug 28 '14 at 01:10
  • Oh, you should include an example of how measuring the exact perimeter of the the British/English/Scottish Island was impossible due to fractals often having infinite perimeters. – Simply Beautiful Art Apr 22 '16 at 21:00
  • The details are a bit hard to see. Could you zoom in on them? – Mark Harrison Sep 11 '21 at 12:58
  • @MarkHarrison To see an amazing zoomed-in version of the Mandelbrot set check out its Wikpedia page, specifically, the video https://en.m.wikipedia.org/wiki/Mandelbrot_set#/media/File%3AMandelbrot_sequence_new.gif. (I am unable to zoom in on the other images.) – user134824 Oct 24 '21 at 20:26
172

This animation shows that a circle's perimeter equals to $2r*\pi$. As ShreevatsaR pointed out, this is obvious because $\pi$ is by definition the ratio of a circle's circumference to its diameter

In this image we can see how the ratio is calculated. The wheel's diameter is 1. After the perimeter is rolled down we can see that its length equals to $\pi$ amount of wheels.

Circle perimeter

Source

totymedli
  • 529
  • 2
  • 11
  • 18
  • 5
    But can you show that a sphere with radius, $r$, has a volume of $V_3(r) = \frac{4}{3} \pi r^3$ and a surface area of $SA_3(r) = 4 \pi r^2$ – Cole Tobin Apr 04 '14 at 16:09
  • 56
    That the circle's perimeter is $2\pi r$ is the definition of $\pi$, so I wouldn't say this is an explanation of the fact; rather it's an illustration of what the definition means, and that the value of $\pi$ is about $3.1$. – ShreevatsaR Apr 05 '14 at 05:24
  • @ColeJohnson There's a neat way to find the volume of a sphere with Cavalieri's principle and Pythagoras. Just look up Cavalieri's principle on Wikipedia, I think it's on there. – Akiva Weinberger Feb 23 '15 at 01:45
  • @ColeTobin Yes, for surface area: https://math.stackexchange.com/a/2398385 – totymedli Nov 16 '21 at 12:11
158

The magnetic pendulum:

the magnetic pendulum fractal

An iron pendulum is suspended above a flat surface, with three magnets on it. The magnets are colored red, yellow and blue.

We hold the pendulum above a random point of the surface and let it go, holding our finger on the starting point. After some swinging this way and that, under the attractions of the magnets and gravity, it will come to rest over one of the magnets. We color the starting point (under our finger) with the color of the magnet.

Repeating this for every point on the surface, we get the image shown above.

Peter
  • 294
  • 1
  • 2
  • 9
144

Simple,visual proof of the Pythagorean theorem. Originally from Pythagorean Theorem Proof Without Words 6).

Pythagorean theorem

Peter Mortensen
  • 627
  • 5
  • 11
Guy
  • 8,531
  • 1
  • 26
  • 52
  • 16
    Perhaps I'm trying to oversimplify, but this visual proof would be way easier (perhaps even trivial) if the legs of the big straight angle (S.A.) triangle (hypotenuse = the circle's diameter, third vertex on the top of that leg of length $\;b\;$) were drawn, and then from basic geometry: " In a S.A. triangle, the height to the hypotenuse divides the triangle in two triangles similar to each other and also to the big triangle". Nicely brought. +1 – DonAntonio Mar 31 '14 at 12:31
  • 6
    Another great pictural proof is 4 triangles in a square: http://www.mathalino.com/sites/default/files/images/01-pythagora.jpg – Sergey Grinev Mar 31 '14 at 15:10
  • @DonAntonio I agree. – Guy Mar 31 '14 at 15:21
  • 7
    I don't get the original equality. Are you relying on the similarity of two triangles? How is that obvious from the diagram? – adam.r Mar 31 '14 at 17:10
  • 62
    It's not immediately obvious to me why $\frac{c+a}{b} = \frac{b}{c-a}$ I'm sure it's very simple and I'll kick myself for asking, sorry. – PeteUK Mar 31 '14 at 17:12
  • 4
    @PeteUK similarity of triangles. Follow the link in my answer. – Guy Mar 31 '14 at 17:35
  • @adam.r see above. – Guy Mar 31 '14 at 17:35
  • 3
    anyone not familiar with Pythagoras' theorem will be unfamiliar with this... – timxyz Apr 03 '14 at 02:43
  • 1
    @timxyz I don't think so. Suppose you had never heard of the pythagorean theorem. You would see this and say "okay cool, $a^2+b^2=c^2$. What's your point?" Granted you might not realize the *significance* of this, but that doesn't mean that you would have problem *accepting* the proof, does it? – Guy Apr 03 '14 at 11:52
  • 11
    Well, I looked at that picture and thought: "Err ... how does this work?" I figured it out, but it was far from obvious for me. – celtschk Apr 03 '14 at 18:33
  • 2
    @celtschk Yes, I guess. As DonAntonio says, two more lines would have made it better. – Guy Apr 03 '14 at 18:34
  • 2
    In my opinion, it's far clearer the way it is. The extra lines would produce a profusion of triangles. The way it is drawn now, there is only one triangle, namely the triangle with sides $a,$ $b,$ and $c.$ The current configuration does, of course, require you to visualize the hypotenuses of the triangles that give rise to the equality, but these hypotenuses themselves play no role since only the lengths of the legs are used. – Will Orrick Apr 06 '14 at 02:28
  • 3
    I agree with DonAntonio. This is not clear at all as is. Without the other triangles this becomes a hunt for meaning instead of a simple *check*, *check*, *check*, **cool**! – Travis Bemrose Apr 13 '14 at 18:34
  • for those who didn't understood it (like me), this is more detailed explanation http://math.stackexchange.com/a/912066/172349 – ihebiheb Aug 28 '14 at 13:40
  • @PeteUK Imagine that a = 0; after that: $$ \frac{c+0}{b} = \frac{b}{c-0} \equiv \frac{c}{b} = \frac{b}{c} \equiv b = c $$ Then c and b are radius of the circle. https://en.wikipedia.org/wiki/Triangle_inequality – swietyy Mar 01 '15 at 12:01
  • 1
    Did anyone else see this as power of a point rather than similarity? – Tae Hyung Kim Jul 25 '15 at 03:34
  • @swietyy For the special case where a term is zero, you can prove many false inequalities to true, where these false and the term is non-zero. a+b = c. Is b==c? – user1998 Aug 27 '17 at 21:10
  • @thkim1011 Yes you can use power of a point in this diagram (if you just extend the half-chord $b$ to a full chord of length $2b$) to prove Pythagoras. By the [intersecting chords theorem](https://en.wikipedia.org/wiki/Power_of_a_point) the products of the line segments of the two intersecting chords are equal so $b \cdot b=(c-a) \cdot (c+a)$ and so $b^2 = c^2 - a^2$ – Silverfish Apr 21 '18 at 11:57
  • 2
    This circle diagram also explains Einstein's formula $E=mc^2$. [I'm sure there is no need to mention that I assume $m=E/c^2$ is already known.] – Michael Jul 10 '19 at 14:39
142

Here's a GIF that I made that demonstrates Phi (golden number)

Phi demonstration

Steve Lewis
  • 101
  • 1
  • 1
  • 2
133

Topology needs to be represented here, specifically knot theory. The following picture is from the Wikipedia page about Seifert Surfaces and was contributed by Accelerometer. Every link (or knot) is the boundary of a smooth orientable surface in 3D-space. This fact is attributed to Herbert Seifert, since he was the first to give an algorithm for constructing them. The surface we are looking at is bounded by Borromean rings.

Seifert surface bounding Borromean rings

Peter Mortensen
  • 627
  • 5
  • 11
N. Owad
  • 6,141
  • 3
  • 19
  • 40
  • 65
    I do not know much about topology so I will take your word for it that this is a beautiful idea/concept. However this picture and your description explain nothing to me. It seems like you missed the "easy to explain" bit in the question. – dfc Apr 07 '14 at 05:25
  • 5
    @dfc I don't know, it seems like you can convey most of the meat here using soap bubbles. – Slade Apr 07 '14 at 20:10
125

Take a look at this great example of Fourier series visualizations written in JavaScript.

Enter image description here

Christian Chapman
  • 4,427
  • 2
  • 22
  • 43
Gandi
  • 101
  • 1
  • 1
  • 2
122

Fourier transform of the light intensity due to a diffraction pattern caused by light going through 8 pinholes and interfering on a wall, for different choices of parameter:

enter image description here

enter image description here

enter image description here

enter image description here

enter image description here

enter image description here

enter image description here

The best thing about them is, they satisfy periodic boundary conditions, and so you can pick one of them and set it as a desktop background by tiling it, resulting in a far more spectacular image than just the single unit cells posted above!

The images seem to be a vast interconnected network of lines once you tile them, but in fact the entire picture is actually just a single circle, which has been aliased into a tiling cell thousands of times.

Here is a video of the first couple thosand patterns: http://www.youtube.com/watch?v=1UVbUWuyNmk

Here is the Mathematica code used to generate and save the images. There are two parameters that are adjustable: mag is the magnification and must be an integer, with 1 generating 600 by 600 images, 2 generating 1200 by 1200 images, etc. i is a parameter which can be any real number between 0 and ~1000, with values between 0 and 500 being typical (most of the preceding images used i values between 200 and 300). By varying i, thousands of unique diagrams can be created. Small values of i create simple patterns (low degree of aliasing), and large values generate complex patterns (high degree of aliasing).

$HistoryLength = 0;
p = {x, y, L};
nnn = 8;
q = 2.0 Table[{Cos[2 \[Pi] j/nnn], Sin[2 \[Pi] j/nnn], 0}, {j, nnn}];
k = ConstantArray[I, nnn];
n[x_] := Sqrt[x.x];
conjugate[expr_] := expr /. Complex[x_, y_] -> x - I y;
a = Table[k[[i]]/n[p - q[[i]]], {i, nnn}];
\[Gamma] = Table[Exp[-I \[Omega] n[p - q[[i]]]/c], {i, nnn}];
expr = \[Gamma].a /. {L -> 0.1, c -> 1, \[Omega] -> 100};
ff = Compile[{{x, _Real}, {y, _Real}}, Evaluate[expr], 
   CompilationTarget -> "C", RuntimeAttributes -> {Listable}];
i = 250;
mag = 1;
d = 6 i mag;
\[Delta] = 0.02 i;
nn = Floor[Length[Range[-d, d, \[Delta]]]/2];
A = Compile[{{x, _Integer}, {y, _Integer}}, Exp[I (x + y)], 
    CompilationTarget -> "C", RuntimeAttributes -> {Listable}] @@ 
   Transpose[
    Outer[List, Range[Length[Range[-d, d, \[Delta]]]], 
     Range[Length[Range[-d, d, \[Delta]]]]], {2, 3, 1}];
SaveImage = 
  Export[CharacterRange["a", "z"][[RandomInteger[{1, 26}, 20]]] <> 
     ".PNG", #] &;
{#, SaveImage@#} &@
 Image[RotateRight[
   Abs[Fourier[
     1 A mag i/
      nnn ff @@ 
       Transpose[
        Outer[List, Range[-d, d, \[Delta]], 
         Range[-d, d, \[Delta]]], {2, 3, 1}]]], {nn, nn}], 
  Magnification -> 1]
DumpsterDoofus
  • 1,570
  • 1
  • 9
  • 19
  • 50
    Those images are incredibly beautiful, but can you explain what exactly they represent and how they were generated? "Fourier transform of the light intensity due to a diffraction pattern caused by light going through 8 pinholes and interfering on a wall" isn't very clear for me. – gregschlom Apr 07 '14 at 09:11
  • 4
    "Visually stunning math concepts *which are easy to explain*" –  Apr 08 '14 at 01:06
  • 4
    @Rahul: It's aliasing of a circle. Aliasing is easy to explain. Draw a big circle on a clear plastic sheet. Cut the image into little squares. Stack the squares on top of each other, and look at it. That's the image. The different images above were done using little squares with various side-lengths. I can post the code if you'd like, there are literally tens of thousands of visually distinct diagrams which can be formed. – DumpsterDoofus Apr 08 '14 at 01:17
  • This reminds me of the Radon transform and the Hough transform. – koo Apr 08 '14 at 23:59
  • 1
    Could you post a link to the code here? – Catherine Hwang Apr 09 '14 at 18:46
  • 1
    > "Here is a video of the first couple thosand patterns: http://www.youtube.com/watch?v=1UVbUWuyNmk" $$$$ Unless you're referring to the finite number of frames in the video, the continuous nature of the real numbers that the radius of the circle comes from, means that video shows the first "several" uncountably infinite patterns. ;-) – Travis Bemrose Apr 13 '14 at 18:54
  • Possibly far too late but worth mentioning: If you create a large image (say mag 2) and then downsize it with a good image software 2 times (office picture manager for example) you could get rid of the aliasing. – Gappy Hilmore Jul 08 '15 at 16:16
  • they have an HR Giger feel to them ... – phdmba7of12 Aug 17 '17 at 00:31
115

Francis Galton's Bean machine is interesting as it demonstrates Central Limit Theorem :

enter image description here

Sil
  • 13,256
  • 3
  • 31
  • 70
107

A very satisfying visualization of the area of a circle.

enter image description here

enter image description here

StopReadingThisUsername
  • 1,523
  • 2
  • 19
  • 37
  • 16
    I like this, but one *tiny* criticism is that it's not visually obvious why the unrolled rings should form a triangle. It's obvious if you can see that the unrolled length of the rings is linearly proportional to their radii, but not visually obvious. – Lqueryvg Sep 01 '14 at 10:52
  • 22
    Is it obvious that you can straighten a ring out and get a rectangle when the ring started out bent? I think a lot of these "bang! pi r squared!" ones leave you with real analysis-induced heebie jeebies :) – msouth Sep 26 '14 at 00:27
  • Kalid Azad and BE are quite amazing. – Hunter Batley Apr 15 '20 at 03:17
100

Math is always fun to learn. Here are some of the images that explain some things beautifully visually

enter image description here enter image description here enter image description here enter image description here enter image description here

Raghavendra
  • 123
  • 1
  • 3
  • 8
  • In E-M wave propagation, I thought the E and M components were supposed to be $90$ degrees out of phase. The figure appears to have them in phase or $180$ degrees out of phase, depending on how you interpret the directions of the axes. – David K Aug 28 '14 at 21:25
  • 1
    @DavidK That's only for circularly polarized photons. This is illustrating a plane-polarized wave, whose components are in phase but offset $90^\circ$. – Mario Carneiro Jan 09 '15 at 01:42
  • @MarioCarneiro Quite right, a quick review of wave equations verify the diagram as shown above. I have this mental image of one of my high school teachers walking across the room waving one arm up and down and one arm side to side to simulate the $E$ and $B$ components, but apparently I don't actually remember it very well after all. – David K Jan 09 '15 at 02:56
  • 14
    When I read the number `142857` it was like a dejavu for me and I was like "OH MY GOED!! I'VE KNOWN THIS NUMBER!" – Qwerty Jan 11 '15 at 04:20
  • yea exactly me too! I remembered it from $\dfrac{1}{7}$ which equals $0.142857..$ and repeats again and again. and $\dfrac{2}{7}$ equals $0.285714$ ! similarly for 3/7 I use this knowledge in many calculations! – Max Payne Apr 21 '15 at 15:08
  • What's the source/explanation on that line-crossing multiplication? That's awesome! – Nate Glenn Jan 30 '16 at 07:59
  • 1
    Magic 9 hiding on it. 142857 -> (1 + 4 + 2 + 8 + 5 + 7 = 27); ( 27 -> 2 + 7 = 9 ) – Brain90 Mar 16 '16 at 10:12
  • 2
    @NateGlenn It's really just a visualisation of [long multiplication](https://en.wikipedia.org/wiki/Multiplication_algorithm#Long_multiplication). (Try writing out the long multiplication of $13 \cdot 12$ and see how it corresponds to the diagram.) – Martin Ender Mar 16 '16 at 15:57
  • Some time ago I asked [a question](https://math.stackexchange.com/q/2460197/415941) about this 142857-number and found numbers with similar poperties. – M. Winter Nov 10 '20 at 19:39
79

One of my favorites - I've seen it somewhere on the web but can't find it again now, so had to reconstruct myself. It is not as pretty but suffices to convey the idea.

                                          enter image description here

It gives good grasp both for $e^x=\lim_{n\to\infty}\left(1+\frac xn\right)^n$ and for $e^{2k\pi i}=1$

Christian Chapman
  • 4,427
  • 2
  • 22
  • 43
  • Another nice visualization of a related problem is Szego's problem of the asymptotic of the roots of the truncated polynomials $P_n(z)$ of the Taylor expansion of $e^z$. – VECH Aug 05 '21 at 08:26
79

I do not know if this meets your criteria of "visually stunning", but nonetheless -

I like this proof of Pythagoras' Theorem (image taken from www.wisfaq.nl):

Pythagoras' Theorem

The key to understanding this is to realize that the inner quadrilateral must be a square - the sides are equal in length (obviously) and each of its angles is $90^{\circ}$ because the two angles on either side sum to $90^{\circ}$, and the sum of the three angles is $180^{\circ}$. The area of this square is $c^2$.

The outer square's area is $(a + b)^2$, which is $c^2$ plus $2 a b$ which is the total area of the four triangles, each of area $\frac{1}{2} a b$.

$(a + b)^2 = c^2 + 2 a b$

$a^2 + b^2 + 2 a b = c^2 + 2 a b$

$a^2 + b^2 = c^2$, which is Pythagoras' theorem.

Aky
  • 597
  • 5
  • 10
62

Visualisation in ancient times: Sum of squares

Let's go back in time for about 2500 years and let's have a look at visually stunning concepts of Pythagorean arithmetic.

Here's a visual proof of

\begin{align*} \left(1^2+2^2+3^2\dots+n^2\right)=\frac{1}{3}(1+2n)(1+2+3\dots+n) \end{align*}

                                       enter image description here

The Pythagoreans used pebbles arranged in a rectangle and linked them with the help of so-called gnomons (sticks) in a clever way. The big rectangle contains $$(1+2n)(1+2+3\dots+n)$$ pebbles. One third of the pebbles is red, two-thirds are blue. The blue thirds contain squares with

$$1\cdot1, 2\cdot2, \dots,n\cdot n$$

pebbles. Dismantling the blue squares into their gnomons shows that they appear in the red part. According to Oscar Becker: Grundlagen der Mathematik this proof was already known to the Babylonians (but also originated from hellenic times).

epi163sqrt
  • 94,265
  • 6
  • 88
  • 219
  • 1
    I found this proof delightful. Is there any free resource checking the claim that it was known to the pythagoreans? – Santropedro Jul 01 '19 at 02:57
  • 3
    @Santropedro: You might find *[this paper](https://www.math.tamu.edu/~don.allen/masters/Greek/pythag.pdf)* interesting (p. 16). – epi163sqrt Jul 01 '19 at 11:13
58

The following animation shows how the surface area of a sphere is calculated.

Surface area calculation for a sphere

totymedli
  • 529
  • 2
  • 11
  • 18
52

This is what happens when you take Pascal's Triangle, and color each entry based on the value modulo 2:

Pascal's Triangle modulo 2

The exact code for this is extremely simple:

def drawModuloPascal(n, p):
    for i in range(0, n + 1):
        print " " * (n - i) ,
        for k in range(0, i + 1):
            v = choose(i , k) % p
            print '\x1b[%sm ' % (';'.join(['0', '30', str(41 + v)]), ) ,
        print "\x1b[0m" # reset the color for the next row

Just provide your own choose(n, r) implementation. The image above is a screenshot of drawModuloPascal(80, 2).

You can also do this modulo other primes, to get even more remarkable patterns, but then it becomes much less "easy to explain."

Adrian Petrescu
  • 1,295
  • 11
  • 27
52

Divergent Series can be visual:

enter image description here

from the Wikipedia

showing that $(1-1+1-1+\dots)^2=1-2+3-4+\dots$

Simply Beautiful Art
  • 71,916
  • 11
  • 112
  • 250
44

This is from betterexplained.com. It's a really cool website with lots of intuitive explanations of maths concepts. This helped me understand Pythagoras' theorem. Actually my go-to website for intuitive explanations of concepts.

Pythagoras' theorem

These are similar triangles. This diagram also makes something very clear:

Area (Big) = Area (Medium) + Area (Small) Makes sense, right? The smaller triangles were cut from the big one, so the areas must add up. And the kicker: because the triangles are similar, they have the same area equation.

Let's call the long side c (5), the middle side b (4), and the small side a (3). Our area equation for these triangles is:

Area = F * hypotenuse^2

where F is some area factor (6/25 or .24 in this case; the exact number doesn't matter). Now let's play with the equation:

Area (Big) = Area (Medium) + Area (Small)

F c^2 = F b^2 + F a^2

Divide by F on both sides and you get:

c^2 = b^2 + a^2

Which is our famous theorem! You knew it was true, but now you know why.

This explains the product rule:

betterexplained

Kevin
  • 289
  • 1
  • 4
  • 11
  • This depends on the idea that the area of all shapes increases on the order of the square of a scale parameter. It's true but it takes a lot of machinery to prove if you don't already believe it to start with. – Christian Chapman Aug 11 '16 at 05:04
  • 2
    @enthdegree That's actually not true. It's true for well behaved 2D shapes (and in fact you can use this to define a type of dimension) but in general it's a factor of $(\Delta x)^d$. I say "well behaved shapes" because when you use this with fractals, you get fractions! – Stella Biderman Apr 06 '17 at 14:06
43

enter image description here

This is @Blue's very nice visual proof from trigonography.com that

$$x+\frac{1}{x}\;\geqslant\; 2$$

Two more illustrations from http://www.doubleroot.in

We see $(x+(1/x))^2 \geq 4$:

Obviously Area of a square, [x+(1/x)]²≥ 4 so..

We know the hypotenuse is always the longest side of a triangle: As we know hypotenuse is always longest side of a triangle.

quid
  • 40,844
  • 9
  • 59
  • 101
  • 5
    BTW: It's cool (and surprising) that you know me from trigonography.com and *not* here. Maybe I should update the site more often, now that I know I have an audience. :) – Blue Apr 09 '16 at 01:12
  • 1
    @Blue Yes, everyone likes a good diagram (a lot) –  Apr 09 '16 at 01:13
  • 3
    hey, i saw you did some editing to remove my watermark there. but please cite the source atleast? www.doubleroot.in or facebook.com/doubleroot – yomayne Aug 14 '18 at 11:05
  • 3
    @yomayne the original poster is not active anymore, thus there was no reply initially. I added a link to the site you mentioned; since by now you passed 100 points you can edit Community Wiki posts. Please add more precise reference as you see fit. (It seems the site is under a CC licence, so I suppose you it is fine that it is reproduce in principle; if not we can also remove it entirely.) – quid Aug 27 '18 at 15:08
38

Ulam Spiral:

enter image description here

Discovered by Stanislaw Ulam, the Ulam Spiral or the Prime Spiral depicts the certain quadratic polynomial's tendency to generate large number of primes.Ulam constructed the spiral by arranging the numbers in a rectangular grid . When he marked the prime numbers along this grid, he observed that the prime numbers thus circled show a tendency to occur along diagonal lines. A 150x150 Ulam Spiral is shown below where the dots represent the occurance of prime numbers. The high density along the diagonal lines can be seen as represented by the darker shade of blue. enter image description here

36

I recently find some stunning visualizations. I preferred to share them all:


$5)$ Mean inequalities [from Proof without words]

enter image description here


$4)$ Streographic projection [by H.Segerman]

enter image description here


$3)$ Farey-Ford Tessellation in non-euclidean geometry [by F.Bonahon]

enter image description here


$2)$ Steiner porism [by Wikipedia]

enter image description here


$1)$ Polynomial roots [by J.Baez]

enter image description here


Aren't them Incredible??

MR_BD
  • 5,261
  • 5
  • 41
  • 59
33

The Julia set of a complex number $c$ is a fractal (for each $c$ you have one) that has a weird property: they visually look like the Mandelbrot set around that point $c$. This becomes clear in this illustration I made for a school project, which consists of tiny images of Julia sets:

mosaic

Magically the Mandelbrot set appears...

Jens Renders
  • 4,133
  • 19
  • 29
  • 9
    Mandelbrot defined his Set as the set of values for which the Julia set is connected; its formula is intimately related to that of a Julia set. – Anton Sherwood Jun 21 '16 at 23:46
31

The sum of the first $n$ squared numbers:

enter image description here

The first 3 triangles are the same, just rotated. Also, notice that

$$\begin{align}1^2&=1\\2^2&=2+2\\3^2&=3+3+3\\\vdots\ \ &\quad\ \ \vdots\qquad\qquad\quad\ddots\\n^2&=n+n+\dots+n+n\end{align}$$

Which is the first triangle. The last triangle is given by $\frac12[n(n+1)(2n+1)]$

Thus,

$$3(1^2+2^2+3^2+\dots+n^2)=\frac{n(n+1)(2n+1)}2$$


Please see here for the original post

and here for a more indepth explanation.

Simply Beautiful Art
  • 71,916
  • 11
  • 112
  • 250
30

It's not exactly stunning, but it is interesting and visual and simple enough for an elementary school child:

There are only 5 platonic solids.

Numberphile has a great video explaining it: https://www.youtube.com/watch?v=gVzu1_12FUc

In short, the reason is that there are only enough space for 3, 4, or 5 equilateral triangles at a corner; only enough space for 3 squares at a corner; and only enough space for 3 pentagons at a corner; and not even enough space for 3 hexagons at a corner, so there are only 5.


Although I guess it was stunning enough for the ancient Greeks to decide that they were the geometric basis of the five elements of the universe: earth, fire, wind, water, aether.

DanielV
  • 22,195
  • 5
  • 35
  • 67
28

How to convert a function from Cartesian to Polar coordinates:

enter image description here

anonymous
  • 2,333
  • 18
  • 42
  • This is brilliant! We're reflecting in $y=x$ line as we are changing from second variable (y) to first variable (r). – cosmo5 Sep 27 '20 at 12:17
  • This seems relevant - https://math.stackexchange.com/a/3806702/308392 So we reflect about y = x to find x = g(y) from y = f(x). Now the distance from the origin to the points on the curve is r, if you look at the cartesian plot, as $\theta$ increases we should see r increase. But in your polar plot, we see minimal $r$ appearing periodically. This does not make sense. I don't see how straight grid lines in the cartesian plot become r = constant lines in the polar plot. I think this is wrong. I think if you use the right transformation it will be - https://www.desmos.com/calculator/q4gmpdkcw2 – Aditya P Mar 13 '21 at 09:00
26

The beauty of watching graphs being constructed has always mesmerized me; I love how such simple figures can be used to make such complicated pictures. And it's especially satisfying with polar graphs.

enter image description here enter image description here

Even simpler things like conic sections:

enter image description here

enter image description here

This one might not be as easy to explain, but brachistochrones are wonderful things.

enter image description here

Robert Howard
  • 2,635
  • 3
  • 19
  • 46
25

This one $($via Proof Without Words$)$ is wonderful but not immediately obvious. Ponder on it and you'll find out how fantastic it is when you get it. enter image description here


Explanation: enter image description here
Set the radius to be $1$, then $$HK=2HI=2\cos\frac{\pi}{7}$$ $$AC=2AB=2\cos\frac{3\pi}{7}$$ $$DG=2DF=-2\cos\frac{5\pi}{7}$$ So $$\begin{align} 2(\cos\frac{\pi}{7}+\cos\frac{3\pi}{7}+\cos\frac{5\pi}{7})&=HK+AC-DG\\ &=HK-(DG-AC)\\ &=HK-(DG-DE)\\ &=HK-EG\\ &=HK-JK\\ &=HJ\\ &=LO\\ &=1 \end{align}$$
MR_BD
  • 5,261
  • 5
  • 41
  • 59
Vim
  • 13,065
  • 4
  • 25
  • 71
  • I don't get it. I see a circle divided into 14 segments and 3 oddly placed parallelograms. Explain this please. – Nick Feb 22 '15 at 07:17
  • @Nick I'll add it in my answer. – Vim Feb 22 '15 at 07:34
  • @Nick I've finished. You can take a look at it if you need it. – Vim Feb 22 '15 at 07:45
  • @Vim how do we know that, for example, AD and CE are parallel? – Joffan Mar 14 '15 at 14:56
  • @Joffan you can calculate the relevant angles, which won't be hard. – Vim Mar 14 '15 at 17:42
  • @Vim I can see that OJ is parallel to HL, but I can't see the other two. I'm not saying they aren't parallel; just that it isn't readily apparent. – Joffan Mar 14 '15 at 18:37
  • @Joffan triangle LOH is isosceles and the angle at O measures $\pi/7$, which means the base angles both measure $3\pi/7$. So LH and OJ both make an angle $3\pi/7$ with the horizontal, and are therefore parallel. – Will Orrick Dec 06 '15 at 09:19
  • Triangles EAD and EAC are congruent by SAS. Since AC and DE are parallel, the interior angles, EAC and AED are congruent. So then AED is also congruent to EAD, and so triangles EAD and EAC are isosceles, from which it follows that quadrilateral ACED is a rhombus. Also, since angle OAD measures $3\pi/7$, triangles EDA and ECA are both similar to triangle LOH. – Will Orrick Dec 06 '15 at 09:41
  • The lines through EG and OD both make an angle of $3\pi/7$ with the line through OE, and are therefore parallel. To see that the line through JE is also parallel to these, examine the quadrilateral ODEJ. Angles JOD and EDO both measure $2\pi/7$, and line segments OJ and DE both have the same length as line segment LH. From this it follows that the line through JE is parallel to the line through OD. – Will Orrick Dec 06 '15 at 09:42
  • 13
    To my mind, however, this proof makes $$\cos\frac{\pi}{7}+\cos\frac{3\pi}{7}+\cos\frac{5\pi}{7}=\frac{1}{2}$$ seem more magical than it really is. One can instead see that the statement is equivalent to $$\cos\frac{\pi}{7}+\cos\frac{3\pi}{7}+\cos\frac{5\pi}{7}+\cos\frac{7\pi}{7}+ \cos\frac{9\pi}{7}+\cos\frac{11\pi}{7}+\cos\frac{13\pi}{7}=0,$$ which expresses that the mean horizontal coordinate of seven equally spaced points on the unit circle is $0$, a fact that follows from rotational symmetry. – Will Orrick Dec 06 '15 at 10:01
25

Polynomials can describe geometric objects

In high school we learn that some low order polynomials can describe geometric shapes:

Basic shapes we all recognize ( as intro ) $$\begin{array}{llr}y&=kx+m& (\text{ line })\\r^2 &= x^2+y^2 & (\text{ circle })\\y &= x^2+ax+b &( \text{ parabola })\end{array}$$

Cool properties consider the rotation $$\left[\begin{array}{c}x\\y\end{array}\right] = \left[\begin{array}{rr}\cos(\phi)&\sin(\phi)\\-\sin(\phi)&\cos(\phi)\end{array}\right]\left[\begin{array}{c}x_{new}\\y_{new}\end{array}\right]$$ and then we substitute each $x^ay^b$ and carry out the multiplications and we will still have a polynomial. By similar reasoning we can do scaling and translation and still remain a polynomial. If we rewrite the polynomials to be expressions equal to 0: $$p_a(x,y)= 0, p_b(x,y) = 0$$ then we can multiply them and use the fact that $$b\cdot a = a\cdot b = 0, \forall a \neq 0, \text{iff } b=0$$ This gives us ability to combine shapes into one and the same representation. We can also do something of the opposite: equation systems which can get the intersection. Example is intersection of two lines is an equation system of two lines. The interesection of a sphere and a plane is a point or a circle. This is also where the expression conic section comes from: an intersection between a cone and something!

And still after all this which is so visually accessible and easy to explain in one sense, still involves challenges in modern math of algebraic geometry has had lots of development even in the last 50 years.

below: $ax^p + by^p - k^p = 0$ for $p=6$. When $p$ grows it will get closer and closer to a rectangle. To the right is the "fifth heart curve" (source: Wolfram Alpha) is an 8th degree polynomial.

mathreadler
  • 24,082
  • 9
  • 33
  • 83
25

The (otherwise also easy to prove) fact that $\sum_\limits{k=1}^n k=\frac{n(n+1)}{2}$ in one picture:

enter image description here

Source of the picture

The Phenotype
  • 5,129
  • 9
  • 22
  • 34
24

This one is only visually stunning in your imagination, but I like it. The derivative of a circle w.r.t. the radius is the circumference. $$\frac{d}{dr}\pi r^2=2\pi r$$ The derivative of a sphere w.r.t. the radius is the area. $$\frac{d}{dr}\frac{4}{3}\pi r^3=4\pi r^2$$ The derivative of a 4-dimensional sphere w.r.t. the radius is the 3-dimensional area. $$\frac{d}{dr}\frac{1}{2}\pi^2 r^4=2\pi^2 r^3$$ This works because the radius is invariant in n-dimensional spheres. Holding a circle, a sphere or a hypersphere requires your hands to be the same distance apart.

atomteori
  • 578
  • 4
  • 11
  • 7
    Also, this isn't limited to circles/spheres at all. It holds for squares and cubes and tetrahedra and possibly other shapes, if the formulas are expressed in terms of the radius of the inscribed circle/sphere. – user3932000 Mar 09 '17 at 02:00
  • 1
    @user3932000 Pls write an answer expanding your comment – dushyanth Nov 19 '17 at 15:25
23

Allow me to join the party guys...

This is another proof of the Pythagorean theorem by The 20th US President James A. Garfield.

enter image description here

A nice explanation about Garfield's proof of the Pythagorean theorem can be found on Khan Academy.

Tunk-Fey
  • 24,769
  • 9
  • 78
  • 106
21

A theorem that I find extraordinarily beautiful and intuitive to understand is Gauss' Theroma Egregium, which basically says that the Gaussian curvature of a surface is an intrinsic property of the surface. Implications of this theorem are immediate, starting from the equivalence of developable surfaces and the 2D euclidean plane, to the impossibility of mapping the globe to an atlas. Wikipedia also provides the common pizza eating strategy of gently bending the slice to stiffen it along its length, as a realization

surajshankar
  • 141
  • 5
  • Sounds interesting but I found that link really impenetrable, as a lay person. – Paul Apr 03 '14 at 13:17
  • Yeah, great theorem, bad example ;) – Carsten S Apr 03 '14 at 21:10
  • 3
    A minor correction: strictly speaking, Gaussian curvature is not a **topological** invariant. – Michael Apr 03 '14 at 22:48
  • 1
    @Michael: Yes, you are right. My mistake. I instead had in mind the surface integral of the gaussian curvature over a closed surface, which _is_ a topological invariant (basically the euler characteristic) – surajshankar Apr 05 '14 at 04:59
19

I've built a bunch of interactive explorations over at Khan Academy. A few of my favorites are:

  • Derivative intuition. Particularly amazing is seeing how $\frac{d}{dx}e^x=e^x$. (Do a few and it should pop up).

  • Exploring mean and median. Light bulbs are twice as likely to burn out before the average lifetime printed on the package. If that statement surprises you, this exploration points out that mean and median aren't the same thing.

  • Exploring standard deviation. Standard deviation is a term that gets thrown around a lot. Play around with this to get a more intuitive sense of what it means.

  • One step equation intuition. Basic introduction to why you can do the same thing to both sides of an equation to solve it.

Peter Mortensen
  • 627
  • 5
  • 11
eater
  • 2,923
  • 2
  • 14
  • 7
18

I would like to add some explorations of the concept asked by the OP of my own:

  1. Visualization of the set of real roots of quadratic equations $ax^2+bx+c=0$, for the specific values of the intervals $a \in [-a_i,a_i]$, $b \in [-b_i,b_i]$, $c \in [-c_i,c_i]$, $a,b,c \in \Bbb N$.

By Cartesian coordinates $(x,y)=(x_1,x_2)$. E.g. $a_i,b_i,c_i=75$:

enter image description here

By Polar coordinates $(\theta, r)=(x_1,x_2)$. E.g. $a_i,b_i,c_i=75$:

enter image description here

Due to the symmetries the opposite patterns $(x,y)=(x_2,x_1)$ and $(\theta,r)=(x_2,x_1)$ are similar.

  1. The Chaos Game on the metric space $S^{1} \times [0,\infty)$ with the metric $d((\theta_1,x_1),(\theta_2,x_2)) = d_{S^1}(\theta_1,\theta_2) + |x_1-x_2|$. The distance in $S^1$ is given by the smallest angle measure between $\theta_1$ and $\theta_2$ (this is actually a scaled Euclidean metric on the unit circle itself).

In this version, the points are $(\theta, r)$, (the angle in radians and the radius). And the three attractor points are $A=(0,0)$,$B=(\frac{5\pi}{4},1)$ and $C=(\frac{7\pi}{4},10^4)$.

enter image description here

This is another example locating the attractor points in the same axis: $A=(0,0)$,$B=(\pi-\frac{\pi}{8},1)$ and $C=(\frac{7\pi}{4},10^4)$.

enter image description here

  1. Contruction step by step of the Voronoi diagram of the points generated by a classic Chaos Game Sierpinski gasket.

enter image description here

enter image description here

  1. And my favorite so far, visualization of the $4$-tuples of the extended Euclidean algorithm in a four dimensional tesseract. The projection of the four dimensional points are shown into a $3D$ visualization adding as a reference a tesseract or hypercube:

enter image description here

enter image description here

iadvd
  • 8,645
  • 11
  • 31
  • 68
  • @Leila you are welcome, if you want to know more about them, I have linked the original questions on each point. – iadvd Apr 06 '17 at 08:16
18

The fact that the graph of inverse of a function is nothing more than its image in line $y=x$ but still finding inverse is so difficult is a math concept I really find amazing.enter image description here

Also inverse of some functions have special name and are really special and useful.

Abhishek Choudhary
  • 708
  • 10
  • 31
18

enter image description here I made this earlier this year in Blender after having spent a few days trying to think of a visual proof of $a^3-b^3 = (a-b)(a^2+ab+b^2)$ so that I could make myself a nice paperweight. I think it's quite clear but I'll explain it anyways,

When it's put together you see the cube $a^3$ with the piece $b^3$ cut out of it. This lets you recognize that each block has a dimension of $(a-b)$ somewhere, so then I pull the pieces apart and lay them next to each other.

When it's laid down you can see how all of them have the same height $(a-b)$ with the red block having a base area of $a^2$, the blue block having a base area of $b^2$ and the green block having a base area of $ab$.

So that shows how $a^3-b^3 = (a-b)a^2 + (a-b)b^2 + (a-b)ab$, which is nicer factored as,

$$a^3-b^3 = (a-b)(a^2+ab+b^2)$$

18

Transpose of a matrix column, this gift shows the easiest proof ever made

enter image description here

L F
  • 3,411
  • 1
  • 20
  • 46
18

There's also some really cool art in Polynomiography. Dr. Bahman Kalantari seems to have made really interesting visualizations of polynomials, and considering these functions are everywhere, it might be cool to check them out.

Polynomiography

A Polynomial

jrab227
  • 1
  • 2
17

A connection between Mathematics and Love: the story goes that a very shy mathematician had fallen in love with a girl but did not dare to tell her. In stead he wrote her a letter with only the following formula: $$y=\pm \sqrt{25-x^2} -\frac{3}{|x|+1}$$ If she was really interested he counted on her drawing the graph of the formula. How the story ended no one knows ...

So how does one construct such a formula? The first part of the formula (the square root without the fraction) is an ellipse: $(3x)^2+(5y)^2=15^2$. Now to get a heart shape the top and bottom of this ellipse must be somewhat lowered and this is accomplished by adding the fraction which is really an adjusted orthogonal hyperbola, $y=\frac{1}{x}$, in such a way that it connects to the ellipse. Desmos is a fantastic tool to illustrate this and hence explaining graphs, functions and and a bit of analytical geometry.

Heart

Nicky Hekster
  • 42,900
  • 7
  • 54
  • 93
15

This site looks very interesting to learn about algebraic surfaces.

Imagem

http://touch-geometry.karazin.ua/list

LCarvalho
  • 253
  • 3
  • 13
15

One of my favourites is from Littlewood's Miscellany, where he amicably mentions that "for the professional the only proof needed" for the one-dimensional fixed point theorem is the following figure. The theorem is:

If $f:[0,1]\rightarrow [0,1]$ is continuous and increasing then, under iteration of $f$, every point is either a fixed point or else converges to a fixed point.

enter image description here

Arbutus
  • 2,223
  • 11
  • 24
15

Riemann integration has always amazed me. Its simple yet extraordinary. enter image description here

Pallavi
  • 432
  • 3
  • 18
15

The surface obtained by spinning a cube on two diametrically opposite corners:

$\hskip{4.5cm}$enter image description here

All the surfaces are ruled surfaces. The top and bottom are simply conical caps. The curved part in the middle is part of a hyperboloid of one sheet. It can be obtained by taking a cylinder of radius and height $\sqrt{\frac23}$ times the edge length of the cube and giving one of the ends a $60^{\large\circ}$ twist.

robjohn
  • 326,069
  • 34
  • 421
  • 800
14

Gluing two Mobius strips along their edges is a Klein bottle.

enter image description here

Source

C.F.G
  • 7,854
  • 2
  • 18
  • 49
14

Just wanted to point out that The Book of Numbers has a lot of the examples above $($ as well as many others $).$

MR_BD
  • 5,261
  • 5
  • 41
  • 59
13

In plane geometry Morley’s theorem is a stunning fact in my opinion: In any triangle, the points of intersections of adjacent trisectors of the angles form an equilateral triangle :enter image description here

In analytical geometry: The generalization of triangles or tetrahedra in n-dimensions is simpleces. And the formula for the simplex volume is a beauty, for example, the volume of four-dimensional simplex which is called pentachoron, pentatope or 5-cell (using the coordinates of its vertices): $$\text{Four dimensional volume} = \pm\frac{1}{4!}\;\begin{vmatrix} \;x_1-x_5 && y_1-y_5 && z_1-z_5 && w_1-w_5\;\\ \;x_2-x_5 && y_2-y_5 && z_2-z_5 && w_2-w_5\;\\ \;x_3-x_5 && y_3-y_5 && z_3-z_5 && w_3-w_5\;\\ \;x_4-x_5 && y_4-y_5 && z_4-z_5 && w_4-w_5\;\\ \end{vmatrix}$$ In the case of triangles we get $$A=±\frac{1}{2}\;\begin{vmatrix} \;x_1-x_3 && y_1-y_3\;\\ \;x_2-x_3 && y_2-y_3\;\\ \end{vmatrix}$$

Of course, we can write it even shorter if we use vectors.

And 4-dimensional spheres are truly amazing, as are tesseracts

And some things are truly arcane (Ramanujan summation formula): $$1+2+3+4+5\,+\,...= -\frac{1}{12}$$ You can get a bit of information about it in this and this Wikipedia articles, but divergent series like this (please refer to Wikipedia again) are not an elementary topic to easily understand. (It's usually met with misunderstanding and downvotes. It's currently 'attacked' in Wikipedia because it is not understood by lay mathematicians. The simplest way to get at least some sort of idea is probably to treat it (it's not completely arbitrary but actually one of the most beautiful concepts in math) as an abstraction, something abstract like, say, square roots of negative numbers. That's a bad comparison, but Ramanujan's idea is slightly tougher than pentachorons or tesseracts. It's not arbitrary assigning $-1/12\,$ to the zeta function. This arcane formula found its way into physics (Casimir force). Euler is considered the first to derive this formula more than two hundred years ago.

As to calculus, my vote would go to the beauty of Euler’s formula already posted in this thread ($\,\boldsymbol{e^{ix}=\cos{x}+i\sin{x}}\,$)

Ken Draco
  • 671
  • 4
  • 10
13

Proof that the area of a circle is $\pi r^2$ without words: Proof Without Words: The Circle

MR_BD
  • 5,261
  • 5
  • 41
  • 59
  • but he has to prove $C=2\pi r$ first – zinking Apr 08 '14 at 06:23
  • 4
    @zinking No, $\pi$ is defined to be the constant that goes in that place in that equation to make it hold. However, I was rather dissatisfied with this "proof". There's lots of distortion involved with deforming the area of the circle into the area of the triangle, that one would have to know calculus to understand why the distortion doesn't matter (at which point, why not just calculate the integral). This is more of a memorization technique to remember the formula. – Travis Bemrose Apr 13 '14 at 20:02
12

A visual display that $0^0=1$. The following is a tetration fractal or exponential map with a pseudo-circle shown in orange. The red area is period $1$ and contains $1$. Example is $1^1=1$. The orange pseudo-circle which contains $0$ is period two. Example is $0^0=1, 0^1=0$.

pseudo-circle

MR_BD
  • 5,261
  • 5
  • 41
  • 59
Daniel Geisler
  • 677
  • 4
  • 12
  • 18
    `0^0 = 1` is not even proven, it was just defined that way to make things easier, right? – Michael Apr 04 '14 at 18:59
  • 9
    It depends on the context, but in the grand scheme of mathematics, it is considered undefined. This is because there are completely logical steps that point to it being zero, and equally logical ones that point to it being one. We can't call it both zero and one, so we call it undefined. – recursive recursion Apr 04 '14 at 21:41
  • 1
    Sorry, you are correct in saying that math, on the whole, has no single undisputable answer for $0^0$. I work with combinatorics where it is usually considered that $0^0=1$ because it makes a number of combinatorial sequences more tidy. – Daniel Geisler Apr 04 '14 at 22:31
  • Check out https://cs.uwaterloo.ca/~alopez-o/math-faq/mathtext/node14.html – Daniel Geisler Apr 05 '14 at 07:13
  • 50
    There is one problem with this. I have no idea how you make that image mean what you say it means... Visually stunning? Yes. Easy to explain? Maybe. Explained? No. – daviewales Apr 05 '14 at 12:31
  • 11
    So, er, how does this show that $0^0=1$? – David Richerby Apr 05 '14 at 17:55
  • 10
    $x^0 = 1$ because the multiplicative identity is one, $0^x = 0$ because zero is the multiplicative fixed point. That is, if and only if we never, ever, ever tussle with fractional powers; then and only then exponentiation is shorthand for repeated multiplication and therefore $0^0 = 1$. If we at any point use fractional powers, then exponentiation is shorthand for it's natural definition: $a^b = e^{b \ln a}$ and we all know that $\ln 0$ is undefined. There is no dispute, once we make the assumptions explicit *the problem goes away.* – Kile Damgaard Asmussen Apr 13 '14 at 09:16
  • I agree with @Karl, but it's even simpler than that. We don't even need to introduce fractional powers. Let $a$ and $b$ be numbers in $\Bbb N$ (or $\Bbb Z, \Bbb Q, \Bbb R$ or any set of numbers containing zero). Then $a^0 = 1$ for all $a \ne 0$ and $0^b = 0$ for all $b \ne 0$. $a^b$ is then defined for every choice of $a$ and $b$ except for both $=0$. Since $0^0$ can't have two different values ($0$ and $1$), it's undefined. This answer should be removed because it's an example of a "convincing" argument that leads to a wrong conclusion. – Travis Bemrose Apr 13 '14 at 19:38
  • @DanielGeisler You can never "zoom in" on a point and see it. $0$ may be contained inside the orange circle, but you can create an image that is all the same color except for at one mathematical point (not a pixel), where it has no color (or is colored differently). No matter how much you use an algorithm to mathematically zoom in on that image, you will never see that differently colored point. It's infinitesimally small (size $0$) and there are uncountably infinite colored points surrounding it, infinitesimally close to it. ... – Travis Bemrose Apr 13 '14 at 19:47
  • @DanielGeisler ... Depending on your algorithm, there is a *chance* that it may decide to color a pixel in your display (or dot in the image) determined by that special point, but it can disappear even when zooming in further, because that pixel or dot is also representing uncountably infinite other regularly colored points "near" that special point. – Travis Bemrose Apr 13 '14 at 19:52
  • Whatever this figure illustrates, it depends on the choice of a branch of the logarithm in a complex domain, and thus can not even give a motivation for defining 0^0 – Maxim Leyenson Jan 03 '22 at 01:09
12

A nationwide math contest in Germany recently came up with a task that I found beautiful to explain, because of two points.

  1. You can get an idea, what the proof is, without applying mathematically accurate theory and this intuitive proof is most likely the right way.

  2. At any given point of this intuitive proof, you can chime in and ask yourself: But how would I say this in mathematical terms? When you find these terms, eventually you get the proof you were looking for.

So here you go: Lea gets the task to write down 2014 numbers. These numbers have to fulfill a specification. For every set of three numbers from that whole set, the arithmetic average of these three must also be within the whole set of $2014$ numbers.

Your task is to proof, that Lea has to write down the same number $2014$ times. Every set of 2014 numbers with any variation in it would not fulfill the specifications.

So since we are talking about layman maths here, I'll go with the intuitive way. We have to find a reason, why choosing a set with different numbers would violate the specifications and we have to proof that always taking the same number would not violate them. The later one is rather easy. Take any arbitrary number three times. The arithmetic average will be the same number, which is in your set already. That wasn't too bad, right?

But what about sets with not all the same numbers? We are mathematicians, so we'll just do what we always do: Chop the problem into pieces we can solve. The first piece is where we have two equal numbers and one other number in our set. Let's assume, the single number is bigger than the two equal numbers. What will that do to our arithmetic average? Right, it will be below the middle between the bigger and the smaller number. We can write that arithmetic average down and specifications are ok. But now we have created another set of three numbers. The single, big number (I'll call it a), one of the two equal numbers (that would be b) and the arithmetic average of a, and b (I'll call that one c). So now we would have to also add the arithmetic average of a, b and c. A quick sketch will show you, that this new number is also slightly below the middle between a and b.

And like that we will always have to add a new number. The arithmetic average of a, b and the new number will never reach the middle. Something, that you can also verify with a few sketches. So we would have to add infinitely more numbers, but we wanted only 2014. Apparently, no two numbers can be equal.

So what if all numbers are different? There is one special case. Let's call our numbers a, b and c again. If b is equally far away from a and c (so b could be 3, a could be 1, then c would be 5). In that case, b is the arithmetic average. But we have to have 2014 different numbers. As soon as we add a fourth number d, it's spoiled. d could be 7, to be still in a distance of 2 to c, but then the set a, b and d would not contain its own arithmetic average. So we know, that within a set of 2014 numbers, we would have sets of three, where these three numbers don't include their own arithmetic average, no matter what we do.

And now we look back at our idea about the set with two equal numbers. We see: As soon as we have a bigger and a smaller number and the number in between those is not exactly in the middle, we can once again start with our endless construction of arithmetic averages. We always replace the number between the bigger and the smaller one by the arithmetic average of the three and we can never reach the middle, but it will always get closer to the middle (thus be another number).

And as I said, making this proof mathematical will not alter it. It will be all the same, but with more equations and sequences. Since we excluded the option of making anything infinite, it is correct as it stands here. This one made me realize: Proofs are not the miracles or the magic they seemed to be for me during high school. Of course, there are hard proofs (and things you can't proof, there is a proof for that), but often you only have to think clearly and to chop the problem into the right pieces.

MR_BD
  • 5,261
  • 5
  • 41
  • 59
Jenga
  • 1
  • 2
  • 4
    Hmm, I don't think "induction by contradiction" ("contradiction by induction"?) is valid even though our intuition would like it. In other words, you've shown that you can't inductively build a set with the desired property, having smaller sets along the way that also have the property. But this doesn't exclude the possibility that the property might not hold for all smaller sets, yet 2014 is the first time you have a set large enough for the property to hold. – Travis Bemrose Apr 13 '14 at 20:36
  • 12
    Sort the list in non-decreasing order as $a_1, a_2, \ldots, a_{2014}$. Since the average of $a_1$, $a_2$, and $a_3$ is among these, it must equal $a_2$, so $a_2 = a_1 + d$ and $a_3 = a_1 + 2d$ for some $d$. Since the average of $a_2$, $a_3$, and $a_4$ is among the list, is must equal $a_3$. Proceeding in this manner we see that $a_k = a_1 + (k-1)d$ for some $d$. Now note that the average of $a_1$, $a_2$, and $a_4$, which is $a_1 + \frac{4}{3} d$, is also in the list. It follows that $d = 0$. – Daniel McLaury Aug 28 '14 at 01:36
12

Check out the "Proofs Without Words" gallery (animated) here:

http://usamts.org/Gallery/G_Gallery.php

And the related proofs here:

http://www.artofproblemsolving.com/Wiki/index.php/Proofs_without_words

Many of these are similar to the ones already listed here, but you get a bunch in one place.

10

In group theory, "visual" explanation of the group $D_4$ (or sometimes called $D_8$, which is dihedral group of order $8$ or degree $4$) was really exciting to me. Even with some elementary knowledge about groups, if someone defines $D_4$ as $$\langle x, a\ |\ a^4 = x^2 = e, axa = x \rangle$$ it might seem meaningless or too abstract to understand. When I first saw this, I simply thought it as something to memorize. But when I saw the visual explanation, it was a "stunning" moment for me because it was really easy to understand and there was nothing to memorize at all:

First let us take a square $S$ with vertices named as $A, B,C, D$.

enter image description here

Then, if we let the element $a$ rotate this square $90^\circ$ clockwise direction and let the element $x$ flip the square through the first diagonal, i.e. $x=y$ line, we can have all $8$ elements of $D_4$ as the following:

enter image description here

$e$: Identity element. It does nothing to $S$.

$a$: Rotates $S$, $90^\circ$ clockwise direction.

$a^2$: Rotates $S$, $90^\circ$ clockwise direction twice, i.e., rotates $S$, $180^\circ$ clockwise direction.

$a^3$: Rotates $S$, $90^\circ$ clockwise direction three times, i.e. rotates $S$, $270^\circ$ clockwise direction.

$x$: Flips $S$ through its first diagonal, i.e., interchanges $A$ and $C$.

$ax$: Flips $S$ through its first diagonal first, then rotates the flipped square $90^\circ$ clockwise direction.

$a^2x$: Flips $S$ through its first diagonal first, then rotates the flipped square $180^\circ$ clockwise direction.

$a^3x$: Flips $S$ through its first diagonal first, then rotates the flipped square $270^\circ$ clockwise direction.


Here, only thing we need to be careful is that rightmost function is applied first (for example, $ax$ means: first flip, then rotate). Now, we can verify the properties that are mentioned in the definition:

First of all, we have $a^4 = e$ because $a^4$ rotates $S$, $90^\circ$ clockwise direction four times, which means $360^\circ$ clockwise direction, which doesn't change the place of any vertex. So it is as same as the identity element $e$.

Secondly, we have $x^2 = e$ because if we flip $S$ through its first diagonal (interchanging $A$ and $C$) and then flip it again, we get $S$ again, which also corresponds to identity element $e$.

Finally, we have $axa = x$, which can be verified by rotating $S$ first, then flipping it and rotating it again. In the end what we get is as same as flipping $S$, which is done by $x$.

ArsenBerk
  • 12,928
  • 3
  • 22
  • 50
9

$$\sum_{i=1}^{\infty}\frac{1}{x^n}=\frac{1}{x-1}$$

In base $x$, this sum equals to 0.1111111.... and if you multiply it by x-1 you get 0.(x-1)(x-1)(x-1).... which equals to 1.

cosmo5
  • 10,379
  • 2
  • 7
  • 34
Tomer Wolberg
  • 456
  • 3
  • 11
9

An important concept im math is $\infty$: the Bernoulli's lemniscate is very similar to its sign. Here the GIF of its construction from an hyperbole:

enter image description here

The equation is very simple:$$\left ( x^2+y^2 \right )^2=2a^2(x^2-y^2)$$

Matteo
  • 5,940
  • 1
  • 8
  • 27
8

One of my favourite mathematical number is $3$ because

$$3=\sqrt{1+2\sqrt{1+3\sqrt{1+4\sqrt{1+5\sqrt{1+\dots}}}}}$$

Similary we can write

$$2=\sqrt{1+\sqrt{1+2\sqrt{1+3\sqrt{1+4\sqrt{1+5\sqrt{1+\dots}}}}}}$$

It can be obtained by going backward as

$$3=\sqrt 9=\sqrt{1+8}=\sqrt{1+2(4)}=\sqrt{1+2\sqrt{16}} =\sqrt{1+2\sqrt{1+3(5)}} =\dots$$

And it can be written further in the same manner.

Ankit Pushpom
  • 21
  • 1
  • 7
8

There is a bijection between $\mathbb R$ and $\mathbb R^2$. That is: a line and the space have the same cardinality.

You can visualize it with (one of) the Peano curve:

(image taken from Wikipedia)

P.S. We use the fact that there is a bijection between $\mathbb R$ and $[0,1]$

An interesting animation: https://www.youtube.com/watch?v=RU0wScIj36o

irene dovichi
  • 124
  • 3
  • 9
  • 5
    The limit curve is surjective, but, unfortunately, it is **not** injective. However, since there is a simple injection $f:[0,1]\mapsto[0,1]^2$ defined by $f(x)=(x,0)$, the [Schröder-Bernstein Theorem](https://en.wikipedia.org/wiki/Schr%C3%B6der%E2%80%93Bernstein_theorem) says that a bijection exists. – robjohn Sep 18 '20 at 20:32
7

We can visually encode factorizations of numbers using $2$-digit palindromes!

Define $n\times n$ matrices $P_n(k)$ and $N_n$ for $x,y\in[0,n)$ as: (the sum is entry-wise)

$$\begin{align} P_n(k)(x,y)&= \begin{cases} 1, & \text{if }x+nk\text{ is a two digit palindrome in number base }y+n\\ 0, & \text{else} \end{cases}\\ N_n&=\sum\limits_{k=0}^{\infty} P_n(k) \end{align}$$

For example, if we color nonzero elements of $N_n$ blue and the zeroes white, we get the following images:

enter image description here

You can notice that prime numbers will always be "complete blue squares" because they do not have nontrivial factors (divisors).

Other numbers will have patterns that represent their factors (divisors) in some way.

For example, notice that even semi-primes $n=2p$ where $p$ is prime, all have the following look:

enter image description here

Or for example, notice that squares of primes $n=p^2$ have the least details:

enter image description here

On the other hand, numbers with a lot of factors such as factorials like $4!=2\cdot3\cdot4 = 24$ or primorials like $p_3\#=2\cdot 3\cdot 5 = 30$ have much more details (factors), and numbers such as powers of primes like $27=3^3$ and $32=2^5$ have regular fractal-like patterns.

enter image description here

And so on.

I must admit that this is a part of my own question, more precisely, it is a part of the introduction to:

Thing get really interesting if we "look inside" the blue squares that represent the prime numbers. But, that part is no longer "easy to explain". (See the link I listed above for more information.)


This is "easy to explain" because:

Factorizations of positive integers $n$ are related to $2$-digit palindromes. That is, if $n$ can be factorized as $n=p\cdot q$ where $p\lt q-1$, this means that $n$ is a $2$-digit palindrome in the number base $q-1$, which we write as:

$$ n=p q=p(q-1)+p=(p, p)_{q-1}, p<q-1, $$

where $(p,p)$ are digits of $n=pq$ in the number base $q-1$.

Vepir
  • 11,937
  • 4
  • 27
  • 78
6

"Cantor's Diagonal Argument" is something I find amazingly beautiful. It is not geometry, it is not artistically stunning, but it visually captures an implausible mathematical truth. It allows you to prove that there are as many rational numbers as natural numbers.


The example below uses two of the simplest infinite sets, that of natural numbers, and that of positive fractions. The idea is to show that both of these sets, ${\mathbb {N} }$ and $\mathbb Q^{+}$ have the same cardinality.

First, the fractions are aligned in an array, as follows: \begin{array}{cccccccccc}{\frac {1}{1}}&&{\tfrac {1}{2}}&&{\tfrac {1}{3}}&&{\tfrac {1}{4}}&&{\tfrac {1}{5}}&\cdots \\&&&&&&&&&\\{\tfrac {2}{1}}&&{\tfrac {2}{2}}&&{\tfrac {2}{3}}&&{\tfrac {2}{4}}&&{\tfrac {2}{5}}&\cdots \\&&&&&&&&&\\{\tfrac {3}{1}}&&{\tfrac {3}{2}}&&{\tfrac {3}{3}}&&{\tfrac {3}{4}}&&{\tfrac {3}{5}}&\cdots \\&&&&&&&&&\\{\tfrac {4}{1}}&&{\tfrac {4}{2}}&&{\tfrac {4}{3}}&&{\tfrac {4}{4}}&&{\tfrac {4}{5}}&\cdots \\&&&&&&&&&\\{\tfrac {5}{1}}&&{\tfrac {5}{2}}&&{\tfrac {5}{3}}&&{\tfrac {5}{4}}&&{\tfrac {5}{5}}&\cdots \\\vdots &&\vdots &&\vdots &&\vdots &&\vdots &\\\end{array} And now the numbers in this construction can be counted as follows, leaving out the fractions which can be simplified.

\begin{array}{lclclclclc}{\tfrac {1}{1}}\ _{\color {Blue}(1)}&{\color {MidnightBlue}\rightarrow }&{\tfrac {1}{2}}\ _{\color {Blue}(2)}&&{\tfrac {1}{3}}\ _{\color {Blue}(5)}&{\color {MidnightBlue}\rightarrow }&{\tfrac {1}{4}}\ _{\color {Blue}(6)}&&{\tfrac {1}{5}}\ _{\color {Blue}(11)}&{\color {MidnightBlue}\rightarrow }\\&{\color {MidnightBlue}\swarrow }&&{\color {MidnightBlue}\nearrow }&&{\color {MidnightBlue}\swarrow }&&{\color {MidnightBlue}\nearrow }&&\\{\tfrac {2}{1}}\ _{\color {Blue}(3)}&&{\tfrac {2}{2}}\ _{\color {Blue}(\cdot )}&&{\tfrac {2}{3}}\ _{\color {Blue}(7)}&&{\tfrac {2}{4}}\ _{\color {Blue}(\cdot )}&&{\tfrac {2}{5}}&\cdots \\{\color {MidnightBlue}\downarrow }&{\color {MidnightBlue}\nearrow }&&{\color {MidnightBlue}\swarrow }&&{\color {MidnightBlue}\nearrow }&&&&\\{\tfrac {3}{1}}\ _{\color {Blue}(4)}&&{\tfrac {3}{2}}\ _{\color {Blue}(8)}&&{\tfrac {3}{3}}\ _{\color {Blue}(\cdot )}&&{\tfrac {3}{4}}&&{\tfrac {3}{5}}&\cdots \\&{\color {MidnightBlue}\swarrow }&&{\color {MidnightBlue}\nearrow }&&&&&&\\{\tfrac {4}{1}}\ _{\color {Blue}(9)}&&{\tfrac {4}{2}}\ _{\color {Blue}(\cdot )}&&{\tfrac {4}{3}}&&{\tfrac {4}{4}}&&{\tfrac {4}{5}}&\cdots \\{\color {MidnightBlue}\downarrow }&{\color {MidnightBlue}\nearrow }&&&&&&&&\\{\tfrac {5}{1}}\ _{\color {Blue}(10)}&&{\tfrac {5}{2}}&&{\tfrac {5}{3}}&&{\tfrac {5}{4}}&&{\tfrac {5}{5}}&\cdots \\\vdots &&\vdots &&\vdots &&\vdots &&\vdots &\\\end{array} Leading to a bijection between $\mathbb{N}$ and $\mathbb{Q^+}$ \begin{array}{cccccccccccccccc}{\color {Blue}1}&{\color {Blue}2}&{\color {Blue}3}&{\color {Blue}4}&{\color {Blue}5}&{\color {Blue}6}&{\color {Blue}7}&{\color {Blue}8}&{\color {Blue}9}&{\color {Blue}10}&{\color {Blue}11}&{\color {Blue}\cdots }\\[3pt]{\color {MidnightBlue}\downarrow }&{\color {MidnightBlue}\downarrow }&{\color {MidnightBlue}\downarrow }&{\color {MidnightBlue}\downarrow }&{\color {MidnightBlue}\downarrow }&{\color {MidnightBlue}\downarrow }&{\color {MidnightBlue}\downarrow }&{\color {MidnightBlue}\downarrow }&{\color {MidnightBlue}\downarrow }&{\color {MidnightBlue}\downarrow }&{\color {MidnightBlue}\downarrow }&{\color {MidnightBlue}\downarrow }\\[3pt]1&{\tfrac {1}{2}}&2&3&{\tfrac {1}{3}}&{\tfrac {1}{4}}&{\tfrac {2}{3}}&{\tfrac {3}{2}}&4&5&{\tfrac {1}{5}}&\cdots \\\end{array} Hence, the cardinality of $\mathbb{Q^{+}}$ and $\mathbb{N}$ is the same. This can be easily extended to all of $\mathbb{Q^{+}}$


Source : https://simple.wikipedia.org/wiki/Cantor%27s_diagonal_argument

SagarM
  • 1,505
  • 12
  • 25
6

A literal meaning of visual math.

Almost any line-drawing can be traced on the cartesian plane using the fourier transformations using only one parametric equation.
In simple terms tracing multiple circles with varying time periods can make a closed curve. Here are a few of my creations (click on the names to open the graph):

  1. Here is Pikachu

(https://www.desmos.com/calculator/wyuubnzylg)

  1. I am Iron Man

Ironman

For a visual proof I suggest watching this piece of art by Grant Sanderson.

Also note that the jittery behaviour and random lines cutting through the figures are not a problem but a rendering issue, which anyways make it look cooler.

P.S. It works as gifts too. You can sketch a portrait using this on a graphing calculator.

Anindya Prithvi
  • 1,143
  • 1
  • 7
  • 28
6

This is a proof of the Pythagorean by US President J. A Garfield.

enter image description here

As you can see, when you line up the triangles like this, it forms a trapezoid. One way to find the area of the trapezoid is by adding up the areas of all the triangles that make it up, so we get $A = \frac{ab}{2} + \frac{ab}{2}+ \frac{c^2}{2}$. Also, we can find the area using the formula for a trapezoid, like this $A = \frac{a+b}{2}*(a+b)$. Now when we set the areas equal to each other, we get $\frac{ab}{2} + \frac{ab}{2}+ \frac{c^2}{2} = \frac{a+b}{2}*(a+b) \implies ab + \frac{c^2}2 = \frac{(a+b)^2}2 \implies 2ab + c^2 = (a+b)^2 \implies a^2+b^2=c^2$

Some Guy
  • 2,620
  • 3
  • 13
  • 28
5

A number spiral of primes with some prime-dense polynomials noted. Alignment of perfect squares is along the right horizontal axis. (The prime-free gap is accounted for by the squares and the squares minus 1.) The pronic numbers are aligned along the left side.

enter image description here

The spiral is plotted counterclockwise like this:

enter image description here

Source: http://numberspiral.com/

michaelmross
  • 156
  • 1
  • 7
  • all swares will be "on line"? and the 3,4,13 branch contains infinitely primes? – L F Sep 25 '20 at 17:09
4

Why is the Möbius strip not orientable?

enter image description here

C.F.G
  • 7,854
  • 2
  • 18
  • 49
4

The heptagonal tiling is a tiling of the hyperbolic plane with heptagons with 3 meeting at each vertex. I once found this image in a Google image search.

enter image description here

It's amazing. It shows that an order 14 triangular tiling has a nice simple relationship with the heptagonal tiling.

Source: https://theinnerframe.org/2018/04/

Timothy
  • 741
  • 7
  • 16
3

I have found it intuitively difficult to see that the sequence $(1+\frac{1}{n})^{n}$ is increasing. However the following picture makes it clear.

enter image description here

We have drawn the graph of the function $\log(1+x)$. It is clear that the graph passes through the origin. By the concavity of the function it is clear that the slope $\log(1+a)/a$ increases as $a$ decreases.

Therefore

$$ \frac{\log(1+\frac{1}{n})}{1/n} < \frac{\log(1+\frac{1}{n+1})}{1/(n+1)} $$

From here the desired result follows.

In fact, one can also see from above that $\lim_{n\to \infty} (1+1/n)^n = e$ since the figure shows that $\log(1+1/n)/(1/n)$ converges to the slope of the tangent at $(0, 0)$, which is $1$.

caffeinemachine
  • 16,806
  • 6
  • 27
  • 107
3

$\Bbb RP^2\sharp\Bbb RP^2\simeq \text{Klein Bottle}$

C.F.G
  • 7,854
  • 2
  • 18
  • 49
3

I just saw this movie on youtube that gives a visual proof that every prime that is 1 modulo 4 can be written as the sum of two squares. This is a highly non-trivial theorem, and that it can be explained visually is impressive.

https://www.youtube.com/watch?v=DjI1NICfjOk

Stephen Montgomery-Smith
  • 25,361
  • 2
  • 29
  • 60
  • See [the MathOverflow answer](https://mathoverflow.net/a/299696/28209) that [Mathologer](https://www.youtube.com/channel/UC1_uAIS3r8Vu6JjXWvastJg)/[Burkard Polster](https://research.monash.edu/en/persons/burkard-polster) was pointed to that led to the creation of the video. – Mark S. Sep 06 '20 at 23:03
3

You might like https://en.wikipedia.org/wiki/Visual_calculus. This was popularised by the beautiful book by Apostol, Mnatsakanian (2013): New horizons in geometry

See also: https://web.archive.org/web/20101211140239/http://www.cco.caltech.edu/~mamikon/VisualCalc.html

enter image description here

asmaier
  • 2,424
  • 17
  • 29
3

Nicomachus's theorem:

enter image description here

Jensen's inequality: enter image description here

Pizza theorem: enter image description here

Golden_Ratio
  • 7,887
  • 3
  • 7
  • 25
2

I don't know if this fits "visually stunning" but it stunned me when I saw it in my 40s because I can't believe no teacher ever explained things this way to me

enter image description here

To me this diagram made so many math concepts clear.

  1. It shows $a = 3$, $b = 4$, $c = 5$

  2. To "Square a number" literally means "turn it into a square". 3 "squared" is a square of 9 units. 4 "squared" is a square of 16 units. 5 "squared" is a square of 25 units.

    No teacher in my life ever showed me this. They just told me something along the lines of "to square a number multiply it by itself"

  3. The "square root" suddenly makes total sense. It's the number before you turned it into a square.

    Again, no teacher ever explained it visually. They just said something like "a square root of a number x is a number y such that $y^2 = x$". That's so much less intuitive than a showing an actual square

  4. It shows an easily understandable example of the pythagorean theorem

    If you take the two smalls sides and turn them into squares, they equal the square of the large side. In the example below 9 + 16 = 25. This is tersely written as $a^2+b^2=c^2$ but was never explained to me visually. It was just something I was told to memorize.

  5. $c = \sqrt{a^2+b^2}$ also makes total sense given the diagram and the realizations above.

I think the diagram could be done better, and maybe with an animation showing each line get built in N units, and then "squared" (turned into a square) but my animation skills are lacking. I posted the SVG here.

gman
  • 243
  • 1
  • 9